SlideShare a Scribd company logo
1 of 51
f ( z ) = u( x , y ) + iv ( x , y ) for z = x + iy
                     f ( z + ∆z ) − f ( z )
 f ' ( z ) = lim [                          ] exists
             ∆z →0            ∆z
Its value does not depend on the direction.
Ex : Show that the function f ( z ) = x 2 − y 2 + i 2 xy is
        differentiable for all values of z .
for ∆z = ∆x + i∆y
                   f ( z + ∆z ) − f ( z )
 f ' ( z ) = lim
             ∆z →0          ∆z
        ( x + ∆x ) 2 − ( y + ∆y ) 2 + 2i ( x + ∆x )( y + ∆y ) − x 2 + y 2 − 2ixy
      =
                                        ∆x + i∆y
                    ( ∆x ) 2 − ( ∆y )2 + 2i∆x∆y
      = 2x + i2 y +
                              ∆x + i∆y
(1) choose ∆y = 0, ∆x → 0 ⇒ f ' ( z ) = 2 x + i 2 y
(2) choose ∆x = 0, ∆y → 0 ⇒ f ' ( z ) = 2 x + i 2 y
* * Another method :
 f ( z ) = ( x + iy ) 2 = z 2
   '             ( z + ∆z )2 − z 2            ( ∆z )2 + 2 z∆z
 f ( z ) = lim [                   ] = lim [                  ]
           ∆z →0        ∆z             ∆z → 0        ∆z
         = lim ∆z + 2 z = 2 z
           ∆z → 0


Ex : Show that the function f ( z ) = 2 y + ix is not
     differentiable anywhere in the complex plane.

f ( z + ∆z ) − f ( z ) 2 y + 2∆y + ix + i∆x − 2 y − ix 2∆y + i∆x
                      =                               =
         ∆z                      ∆x + i∆y               ∆x + i∆y
if ∆z → 0 along a line thriugh z of slope m ⇒ ∆y = m ∆x
                   f ( z + ∆z ) − f ( z )            2 ∆ y + i∆ x    2m + i
 f ' ( z ) = lim                          = lim [                 ]=
             ∆z →0          ∆z             ∆x ,∆y →0 ∆x + i∆y        1 + im
The limit depends on m (the direction), so f ( z )
is nowhere differentiable.
Ex : Show that the function f ( z ) = 1 /(1 − z ) is analytic everywhere
     except at z = 1.

                f ( z + ∆z ) − f ( z )             1     1       1
f ' ( z ) = lim [                      ] = lim [ (            −      )]
        ∆z →0            ∆z                ∆z →0 ∆z 1 − z − ∆z 1 − z
                          1                     1
       = lim [                          ]=
         ∆z →0 (1 − z − ∆z )(1 − z )        (1 − z ) 2
Provided z ≠ 1, f ( z ) is analytic everywhere such that
f ' ( z ) is independent of the direction.
20.2 Cauchy-Riemann relation
 A function f(z)=u(x,y)+iv(x,y) is differentiable and analytic,
 there must be particular connection between u(x,y) and v(x,y)
                f ( z + ∆z ) − f ( z )
L = lim [                              ]
       ∆z →0             ∆z
 f ( z ) = u( x , y ) + iv ( x , y ) ∆z = ∆x + i∆y
 f ( z + ∆z ) = u( x + ∆x , y + ∆y ) + iv ( x + ∆x , y + ∆y )
                       u( x + ∆x , y + ∆y ) + iv ( x + ∆x , y + ∆y ) − u( x , y ) − iv ( x , y )
⇒ L = lim [                                                                                      ]
           ∆x ,∆y →0                                  ∆x + i∆y
(1) if suppose ∆z is real ⇒ ∆y = 0
                    u( x + ∆x , y ) − u( x , y )    v ( x + ∆x , y ) − v ( x , y ) ∂u         ∂v
⇒ L = lim [                                      +i                                 ]=     +i
           ∆x →0                 ∆x                              ∆x                    ∂x     ∂x
(2) if suppose ∆z is imaginary ⇒ ∆x = 0
                    u( x , y + ∆y ) − u( x , y )    v ( x , y + ∆y ) − v ( x , y )        ∂u ∂v
⇒ L = lim [                                      +i                                ] = −i    +
           ∆y →0                i ∆y                            i ∆y                      ∂y ∂y
      ∂u ∂v                  ∂v       ∂u
           =          and         =-        Cauchy - Riemann relations
      ∂x ∂y                  ∂x       ∂y
Ex : In which domain of the complex plane is
         f ( z ) =| x | − i | y | an analytic function?
u( x , y ) =| x |, v ( x , y ) = − | y |
    ∂u ∂v             ∂           ∂
(1)       =     ⇒         | x |= [− | y |] ⇒ (a) x > 0, y < 0 the fouth quatrant
    ∂x ∂y           ∂x           ∂y
                                              (b) x < 0, y > 0 the second quatrant
    ∂v        ∂u        ∂                ∂
(2)       =−       ⇒ [− | y |] = − | x |
    ∂x        ∂y        ∂x               ∂y

z = x + iy and complex conjugate of z is z * = x − iy
⇒ x = ( z + z * ) / 2 and y = ( z − z * ) / 2i
    ∂f       ∂f ∂x ∂f ∂y 1 ∂u ∂v              i ∂v ∂u
⇒        =           +         = ( − )+ ( + )
   ∂z * ∂x ∂z * ∂y ∂z * 2 ∂x ∂y               2 ∂x ∂y
If f ( z ) is analytic , then the Cauchy - Riemann relations
are satisfied. ⇒ ∂f / ∂z * = 0 implies an analytic fonction of z contains
the combination of x + iy , not x − iy
If Cauchy - Riemann relations are satisfied


   ∂ ∂u   ∂ ∂v   ∂ ∂v     ∂ ∂u  ∂ 2u ∂ 2u
(1) ( ) =   ( )=   ( ) = − ( )⇒ 2 + 2 2 = 0
   ∂x ∂x  ∂x ∂y  ∂y ∂x    ∂y ∂y ∂x   ∂ y
                                                 ∂ 2v       ∂ 2v
(2) the same result for function v ( x , y ) ⇒     2
                                                        +   2 2
                                                                   =0
                                                 ∂x      ∂ y
⇒ u( x , y ) and v ( x , y ) are solutions of Laplace' s
 equation in two dimension.

For two families of curves u( x , y ) = conctant and v ( x , y ) = constant,
the normal vectors corresponding the two curves, respectively, are
             ∂u ˆ ∂u ˆ                    ∂v ˆ ∂v ˆ
∇u( x , y ) =    i+    j and ∇v ( x , y ) =    i+    j
              ∂x    ∂y                      ∂x    ∂y
           ∂u ∂v ∂u ∂v     ∂u ∂u ∂u ∂u
∇u ⋅ ∇v =         +       =−          +         = 0 orthogonal
             ∂x ∂x ∂y ∂y     ∂x ∂y ∂y ∂x
20.3 Power series in a complex variable
              ∞               ∞
f (z) =      ∑ an z   n
                          =   ∑ an r n exp(inθ )
             n= 0             n=0
     ∞
if   ∑ | an | r n is convergent ⇒ f ( z ) is absolutely convergent
     n=0
      ∞
Is ∑ | a n | r n convergent or not, can be justisfied by" Cauchy root test".
     n= 0
                                             1
The radius of convergence R ⇒                  = lim | a n |1 / n ⇒ (1) | z |< R absolutely convergent
                                             R n→ ∞
                                                                    (2) | z |> R divergent
                                                                    (3) | z |= R undetermined
       ∞
         zn        1
(1) ∑       ⇒ lim ( )1 / n = 0 ⇒ R = ∞ converges for all z
    n= 0 n!   n→∞ n!
       ∞
(2) ∑ n! z n ⇒ lim ( n! )1 / n = ∞ ⇒ R = 0 converges only at z = 0
      n= 0            n →∞
20.4 Some elementary functions
                         ∞
                     zn
Define exp z = ∑
               n = 0 n!

Ex : Show that exp z1 exp z 2 = exp( z1 + z 2 )
                     ∞
                      ( z1 + z 2 ) n
exp( z1 + z 2 ) = ∑
                  n=0      n!
                      ∞
                          1 n n
                 =   ∑ n!(C 0 z1 + C1n z1n−1 z2 + C 2 z1n−2 z2 + C rn z1n−r z2 + ... + C n z2 )
                                                    n        2               r           n n

                     n= 0
                                                    n
                                        s r        Cr   1     n!         1
set n = r + s ⇒ the coeff. of          z1 z 2   is    =                =
                                                   n! n! ( n − r )! r ! s! r !
                      ∞
                      zs ∞ zr       ∞ ∞
                                               1 s r
exp z1 exp z 2 = ∑ ∑              = ∑∑               z1 z 2
                 s =0 s! r = 0 r ! s =0 r = 0 s! r !
                              s r
There are the same coeff. of z1 z 2 for the above two terms.
Define the complex comonent of a real number a > 0
                                  ∞
                                  z n (ln a ) n
          a = exp( z ln a ) = ∑
            z

                              n=0      n!
(1) if a = e ⇒ e z = exp( z ln e ) = exp z the same as real number
                          iy              y 2 iy 3
(2) if a = e, z = iy ⇒ e = exp(iy ) = 1 −    −     + ...
                                          2!   3!
                                    y2 y4                   y3
                               = 1−   +   + ..... + i ( y −    + ...) = cos y + i sin y
                                    2! 4!                   3!
(3) if a = e , z = x + iy ⇒ e x + iy = e x e iy = exp( x )(cos y + i sin y )
Set exp w = z
Write z = r exp iθ for r is real and − π < θ ≤ π
⇒ z = r exp[i (θ + 2nπ )] ⇒ w = Lnz = ln r + i (θ + 2nπ )
Lnz is a multivalued function of z .
Take its principal value by choosing n = 0
⇒ ln z = ln r + iθ -π < θ ≤ π
If t ≠ 0 and z are both complex numbers, we define
            t z = exp( zLnt )

Ex : Show that there are exactly n distinct nth roots of t .
     1
               1
    tn   = exp( Lnt ) and t = r exp[i (θ + 2kπ )]
               n
     1                                    1
               1         (θ + 2kπ )               (θ + 2kπ )
⇒   tn   = exp[ ln r + i            ] = r n exp[i            ]
               n             n                        n
20.5 Multivalued functions and branch cuts
A logarithmic function, a complex power and a complex root are all
multivalued. Is the properties of analytic function still applied?

Ex : f ( z ) = z 1 / 2 and z = r exp(iθ )                (A)               C
                                                               y

(A) z traverse any closed contour C that
    dose not enclose the origin, θ return                              r
    to its original value after one complete                           θ
                                                                                   x
    circuit.
                                                               (B)         y
(B) θ → θ + 2π enclose the origin
                                                                  C'           r
        1/ 2                     1/ 2
    r          exp(iθ / 2) → r          exp[i (θ + 2π ) / 2]
                                                                               θ
                             = − r 1 / 2 exp(iθ / 2)                               x
    ⇒ f (z) → − f (z)
    z = 0 is a branch point of the function f ( z ) = z 1 / 2
Branch point: z remains unchanged while z traverse a closed contour C
about some point. But a function f(z) changes after one complete circuit.

Branch cut: It is a line (or curve) in the complex plane that we must cross ,
so the function remains single-valued.


                                           y
   Ex : f ( z ) = z 1 / 2
   restrict θ ⇒ 0 ≤ θ < 2π
   ⇒ f ( z ) is single - valued                0
                                                              x
Ex : Find the branch points of f ( z ) = z 2 + 1 , and hence sketch
    suitable arrangements of branch cuts.


f ( z ) = z 2 + 1 = ( z + i )( z − i ) expected branch points : z = ± i
set z − i = r1 exp(iθ1 ) and z + i = r2 exp(iθ 2 )
⇒ f ( z ) = r1r2 exp(iθ1 / 2) exp(iθ 2 / 2)
         = r1r2 exp[i (θ1 + θ 2 )]
If contour C encloses
(1) neither branch point, then θ1 → θ1 , θ 2 → θ 2 ⇒ f ( z ) → f ( z )
(2) z = i but not z = − i , then θ1 → θ1 + 2π , θ 2 → θ 2 ⇒ f ( z ) → − f ( z )
(3) z = − i but not z = i , then θ1 → θ1 , θ 2 → θ 2 + 2π ⇒ f ( z ) → − f ( z )
(4) both branch points, then θ1 → θ1 + 2π ,θ 2 → θ 2 + 2π ⇒ f ( z ) → f ( z )
f ( z ) changes value around loops containing
either z = i or z = − i . We choose branch cut as follows :


 (A)      y
                                              (B)      y
                                                           i
              i

                          x                                     x
              −i
                                                           −i
20.6 Singularities and zeros of complex function
                                             g( z )
Isolated singularity (pole) : f ( z ) =
                                          ( z − z0 ) n
n is a positive integer, g( z ) is analytic at all points in
some neighborhood containing z = z0 and g( z0 ) ≠ 0,
the f ( z ) has a pole of order n at z = z0 .

* * An alternate definition for that f ( z ) has a pole of
    order n at z = z0 is
   lim [( z − z0 ) n f ( z )] = a
  z → z0
   f ( z ) is analytic and a is a finite, non - zero complex number
(1) if a = 0, then z = z0 is a pole of order less than n.
(2) if a is infinite, then z = z0 is a pole of order greater than n.
(3) if z = z0 is a pole of f ( z ) ⇒| f ( z ) |→ ∞ as z → z0
(4) from any direction, if no finite n satisfies the limit ⇒ essential singularity
Ex : Find the singularities of the function
                  1         1
(1) f ( z ) =          −
                1− z 1+ z
                      2z
⇒ f (z) =                         poles of order 1 at z = 1 and z = −1
                (1 − z )(1 + z )
(2) f ( z ) = tanh z
                 sinh z exp z − exp(− z )
              =           =
                 cosh z exp z + exp(− z )
     f ( z ) has a singularity when exp z = − exp(− z )
⇒ exp z = exp[i ( 2n + 1)π ] = exp(− z ) n is any integer
                                          1
⇒ 2 z = i ( 2n + 1)π ⇒ z = ( n + )πi
                                          2
Using l' Hospital' s rule
                 [ z − ( n + 1 / 2)πi ] sinh z                     [ z − ( n + 1 / 2)πi ] cosh z + sinh z
     lim {                                     }=      lim {                                              }=1
z →( n+1 / 2 )πi            cosh z                z →( n+1 / 2 )πi                  sinh z
each singularity is a simple pole (n = 1)
Remove singularties :
 Singularity makes the value of f ( z ) undetermined, but lim f ( z )
                                                               z→ z0
 exists and independent of the direction from which z0 is approached.


Ex : Show that f ( z ) = sin z / z is a removable singularity at z = 0

Sol : lim f ( z ) = 0 / 0 undetermined
     z →0

             1       z3 z5                     z3 z5
     f (z) = (z −        +   − ........) = 1 −   +   − ....
             z       3! 5!                     3! 5!
     lim f ( z ) = 1 is independent of the way z → 0, so
     z →0
     f ( z ) has a removable singularity at z = 0.
The behavior of f ( z ) at infinity is given by that of
 f (1 / ξ ) at ξ = 0, where ξ = 1 / z


Ex : Find the behavior at infinity of (i) f ( z ) = a + bz −2
     (ii) f ( z ) = z (1 + z 2 ) and (iii) f ( z ) = exp z
(i) f ( z ) = a + bz − 2 ⇒ set z = 1 / ξ ⇒ f (1 / ξ ) = a + bξ 2
    is analytic at ξ = 0 ⇒ f ( z ) is analytic at z = ∞
(ii) f ( z ) = z (1 − z 2 ) ⇒ f (1 / ξ ) = 1 / ξ + 1 / ξ 3 has a pole of
     order 3 at z = ∞
                                        ∞
(iii) f ( z ) = exp z ⇒ f (1 / ξ ) =   ∑ (n! )−1ξ −n
                                       n= 0
      f ( z ) has an essential singularity at z = ∞
If f ( z0 ) = 0 and f ( z ) = ( z − z0 )n g ( z ), if n is
a positive integer, and g( z0 ) ≠ 0
(i) z = z0 is called a zero of order n.
(ii) if n = 1, z = z0 is called a simple zero.
(iii) z = z0 is also a pole of order n of 1 / f ( z )
20.10 Complex integral                                     y
                                                                B
A real continuous parameter t , for α ≤ t ≤ β
                                                      C2
x = x ( t ), y = y( t ) and point A is t = α ,
                                                           C1
point B is t = β
⇒ ∫ f ( z )dz = ∫ ( u + iv )(dx + idy )                              x
    C                C
                                                      A         C3
   = ∫ udx − ∫ vdy + i ∫ udy + i ∫ vdx
        C        C          C          C
        β   dx       β dy       β dy       β dx
   =∫ u        dt − ∫ v dt + i ∫ u dt + i ∫ v    dt
        α   dt       α dt       α dt       α  dt
Ex : Evaluate the complex integral of f ( z ) = 1 / z , along
       the circle |z| = R, starting and finishing at z = R.                 y
z ( t ) = R cos t + iR sin t , 0 ≤ t ≤ 2π                                               C1
dx                   dy                          1      x − iy                  R
       = − R sin t ,    = R cos t , f ( z ) =         = 2      = u + iv ,           t
dt                   dt                        x + iy x + y 2
                                                                                             x
           x         cos t         −y          − sin t
u= 2             =         ,v = 2            =
        x + y2         R         x + y2           R
    1       2π cos t                   2π − sin t
∫C1 z dz = ∫0 R (− R sin t )dt − ∫0 ( R ) R cos tdt
             2π cos t                2π − sin t
        + i∫          R cos tdt + i ∫ (         )( − R sin t )dt
            0    R                   0     R
        = 0 + 0 + iπ + iπ = 2πi
* * The integral is also calculated by
        dz     2π − R sin t + iR cos t       2π
    ∫C1 z 0 R cos t + iR sin t
           =∫                          dt = ∫ idt = 2πi
                                             0

The calculated result is independent of R.
Ex : Evaluate the complex integral of f ( z ) = 1 / z along
      (i) the contour C 2 consisting of the semicircle | z |= R in
            the half - plane y ≥ 0
        (ii) the contour C 3 made up of two straight lines C 3a and C 3b
(i) This is just as in the previous example, but for
                                                                                      y
   0 ≤ t ≤ π ⇒ ∫ dz / z = πi                                                         iR
                   C2
                                                                              C 3b
(ii) C 3a : z = (1 − t ) R + itR for 0 ≤ t ≤ 1                                            C 3a
    C 3b : − sR + i (1 − s ) R for 0 ≤ s ≤ 1                        s=1                          t=0
    dz    1    − R + iR             1      − R − iR                       −R                     R x
∫C3 z 0 R + t (− R + iR) 0 iR + s(− R − iR)
       =∫                    dt + ∫                         dt

             1 −1+ i             1    2t − 1              1       1
1st term ⇒ ∫              dt = ∫                 dt + i ∫                 dt
             0 1 − t + it       0 1 − 2 t + 2t 2         0 1 − 2t + 2t 2

              1                             i              t −1/ 2 1
            = [ln(1 − 2t + 2t 2 )] |1 + [2 tan −1 (
                                       0                            )] |0
              2                             2                1/ 2
                   i π         π       πi             a                      x
            = 0 + [ − ( − )] =                  ∫ a2 + x2     dx = tan −1 ( ) + c
                   2 2         2        2                                    a
1  1+ i               1 (1 + i )[ s − i ( s − 1)]
2nd term ⇒ ∫                   ds = ∫                            ds
              0 s + i ( s − 1)       0       2
                                            s + ( s − 1)    2

                1   2s − 1             1      1
           =∫                ds + i ∫                  ds
              0 2s 2 − 2s + 1         0 2s 2 − 2s + 1

             1                                      s −1/ 2 1
           = [ln( 2 s 2 − 2 s + 1)] |1 + i tan −1 (
                                     0                     ) |0
             2                                        1/ 2
                   π       π      πi
           = 0 + i[ − ( − )] =
                    4      4       2
       dz
⇒  ∫C3 z = πi
The integral is independent of the different path.
Ex : Evaluate the complex integral of f ( z ) = Re( z ) along
      the path C1 , C 2 and C 3 as shown in the previous examples.
                  2π
(i) C1 : ∫             R cos t ( − R sin t + iR cos t )dt = iπR 2
               0
                  π                                       iπ 2
(ii) C 2 : ∫ R cos t ( − R sin t + iR cos t )dt =            R
               0                                           2
(iii) C 3 = C 3a + C 3b :
        1                                  1
       ∫0   (1 − t ) R( − R + iR )dt + ∫ ( − sR )( − R − iR )ds
                                           0
              1                                1
   = R 2 ∫ (1 − t )( −1 + i )dt + R 2 ∫ s(1 + i )ds
              0                                0
    1 2             1
   =  R ( −1 + i ) + R 2 (1 + i ) = iR 2
    2               2
The integral depends on the different path.
20.11 Cauchy theorem
If f ( z ) is an analytic function, and f ' ( z ) is continuous
at each point within and on a closed contour C
⇒ ∫ f ( z )dz = 0
       C

   ∂p( x , y )     ∂q( x , y )
If             and             are continuous within and
        ∂x            ∂y
on a closed contour C, then by two - demensional
                                   ∂p ∂q
divergence theorem ⇒ ∫∫ ( + )dxdy = ∫ ( pdy − qdx )
                                R ∂x    ∂y        C

f ( z ) = u + iv and dz = dx + idy
I = ∫ f ( z )dz = ∫ ( udx − vdy ) + i ∫ (vdx + udy )
       C          C                  C
            ∂ ( − u) ∂ ( − v )                ∂ ( − v ) ∂u
     = ∫∫ [         +          ]dxdy + i ∫∫ [          + ]dxdy = 0
         R     ∂y      ∂x                  R    ∂y      ∂x
f ( z ) is analytic and the Cauchy - Riemann relations apply.
Ex : Suppose twos points A and B in the complex plane are joined
     by two different paths C1 and C 2 . Show that if f ( z ) is an
          analytic function on each path and in the region enclosed by the
          two paths then the integral of f ( z ) is the same along C1 and C 2 .

                                                                                    y
                                                                                        B
∫C   1
         f ( z )dz − ∫
                      C2
                           f ( z )dz = ∫
                                            C1 − C 2
                                                       f ( z )dz = 0   C1
path C1 − C 2 forms a closed contour enclosing R                                R

⇒∫            f ( z )dz = ∫     f ( z )dz                                                   x
         C1                C2

                                                                            A
Ex : Consider two closed contour C and γ in the Argand diagram, γ being
sufficiently small that it lies completely with C . Show that if the function
f ( z ) is analytic in the region between the two contours then ∫ f ( z )dz = ∫ f ( z )dz
                                                                                    C             γ

the area is bounded by Γ, and
 f ( z ) is analytic                                                                         C
                                                                                y
∫Γ f ( z )dz = 0                                                                                      γ
                                                                                             C1
             = ∫ f ( z )dz + ∫    f ( z )dz + ∫    f ( z )dz + ∫    f ( z )dz
                C             γ               C1               C2
If take the direction of contour γ as that of                                           C2

contour C ⇒ ∫ f ( z )dz = ∫ f ( z )dz
                  C               γ
                                                                                                          x
Morera' s theorem :
if f ( z ) is a continuous function of z in a closed domain R
bounded by a curve C , for ∫ f ( z )dz = 0 ⇒ f ( z ) is analytic.
                                      C
20.12 Cauchy’s integral formula
 If f ( z ) is analytic within and on a closed contour C
                                               1      f (z)
                                              2πi ∫C z − z0
 and z0 is a point within C then f ( z0 ) =                 dz


          f (z)          f (z)                                   C
I=∫             dz = ∫         dz
     C   z − z0       γ z−z
                             0
for z = z0 + ρ exp(iθ ), dz = iρ exp(iθ )dθ                      γ
      2π     f ( z0 + ρe iθ )                                        z0
I=∫                             iρe iθ dθ
     0           ρe iθ
         2π                          ρ →0
                          iθ
  = i∫        f ( z0 + ρe )dθ = 2πif ( z0 )
         0
The integral form of the derivative of a complex function :
                              1         f (z)
                f ' ( z0 ) =
                             2πi ∫C ( z − z )2 dz
                                           0

                  f ( z 0 + h) − f ( z 0 )
 f ' ( z0 ) = lim
           h→ 0              h
                    1       f (z)         1   1
          = lim [
            h→0 2πi
                         ∫C h z − z0 − h z − z0 )dz ]
                                  (         −

                      1              f (z)
          = lim [
              h→ 0   2πi ∫C ( z − z0 − h)( z − z0 )dz ]
               1         f (z)
          =
              2πi ∫C ( z − z )2 dz
                             0


                                         n!         f (z)
                                        2πi ∫C ( z − z0 )n+1
For nth derivative f ( n ) ( z0 ) =                          dz
Ex : Suppose that f ( z ) is analytic inside and on a circleC of radius
 R centered on the point z = z0 . If | f ( z ) |≤ M on the circle, where
                                                     Mn!
 M is some constant, show that | f ( n ) ( z0 ) |≤          n
                                                                .
                                                        R
                      n!       f ( z )dz    n! M       Mn!
| f ( n ) ( z0 ) |=      |∫              |≤       2πR = n
                      2π C ( z − z0 )n+1 2π R n+1      R

Liouville' s theorem : If f ( z ) is analytic and bounded for all
z then f ( z ) is a constant.

                                                   Mn!
Using Cauchy' s inequality : | f ( n ) ( z0 ) |≤
                                                   Rn
set n = 1 and let R → ∞ ⇒ | f ' ( z0 ) |= 0 ⇒ f ' ( z0 ) = 0
Since f ( z ) is analytic for all z , we may take z0 as any
point in the z - plane. f ' ( z ) = 0 for all z ⇒ f ( z ) = constant
20.13 Taylor and Laurent series
 Taylor’s theorem:
 If f ( z ) is analytic inside and on a circle C of radius R centered
 on the point z = z0 , and z is a point inside C, then
             ∞                          ∞
                                              f ( n ) ( z0 )
  f (z) =   ∑ a n ( z − z0 )   n
                                   =   ∑           n!
                                                             ( z − z0 ) n
            n= 0                       n= 0

                                                                    1     f (ξ )
                                                                   2πi ∫C ξ − z
f ( z ) is analytic inside and on C, so f ( z ) =                               dξ where ξ lies on C

                                                                                      ∞
         1                           z − z0     1     1                                    z − z0 n
expand
       ξ −z
            as a geometric series in
                                     ξ − z0
                                            ⇒      =
                                              ξ − z ξ − z0
                                                                                     ∑(    ξ − z0
                                                                                                  )
                                                                                     n=0

           1      f (ξ ) ∞ z − z0 n        1 ∞                     f (ξ )
⇒ f (z) =                ∑ ( ξ − z ) dξ = 2πi ∑ ( z − z0 )n ∫C (ξ − z )n+1 dξ
          2πi ∫C ξ − z0 n= 0      0           n= 0                    0

               1 ∞               n 2πif
                                        ( n)
                                             ( z0 ) ∞             n f
                                                                      (n)
                                                                          ( z0 )
            =     ∑
              2πi n=0
                      ( z − z0 )
                                        n!
                                                   = ∑ ( z − z0 )
                                                                        n!
                                                     n=0
If f ( z ) has a pole of order p at z = z0 but is analytic at every other point inside
and on C . Then g ( z ) = ( z − z0 ) p f ( z ) is analytic at z = z0 and expanded as a Taylor
                  ∞
series g( z ) =   ∑ bn ( z − z0 )n .
                  n=0
Thus , for all z inside C f ( z ) can be exp anded as a Laurent series
                    a− p         a − p +1               a
        f (z) =             +               + ....... + −1 + a0 + a1 ( z − z0 ) + a 2 ( z − z0 )2
                ( z − z0 ) p ( z − z0 ) p−1            z − z0

                        g ( n ) ( z0 )    1         g( z )
a n = bn+ p    and bn =
                             n!
                                       =
                                         2πi ∫ ( z − z )n+1 dz                      C2
                                                      0
                                                                                        R
        1          g( z )            1         f (z)
       2πi ∫ ( z − z0 ) n+1+ p      2πi ∫ ( z − z0 ) n+1
⇒ an =                         dz =                      dz
                                                                                         z 0 C1
           ∞
f (z) =    ∑ an ( z − z0 )n is analytic in a region R between
          n= −∞
two circles C1 and C 2 centered on z = z0
∞
                     f (z) =    ∑ a n ( z − z0 ) n
                               n = −∞

(1) If f ( z ) is analytic at z = z0 , then all an = 0 for n < 0.
   It may happen a n = 0 for n ≥ 0, the first non - vanishing
   term is a m ( z-z0 ) m with m > 0, f ( z ) is said to have a zero
   of order m at z = z0 .


(2) If f ( z ) is not analytic at z = z0
    (i) possible to find a − p ≠ 0 but a − p− k = 0 for all k > 0
         f ( z ) has a pole of order p at z = z0 , a −1 is called the residue of f ( z )
    (ii) impossible to find a lowest value of − p ⇒ essential singularity
1
 Ex : Find the Laurent series of f ( z ) =             3
                                                           about the singularities
                                         z ( z − 2)
 z = 0 and z = 2. Hence verify that z = 0 is a pole of order 1 and z = 2 is a
 pole of order 3, and find the residue of f ( z ) at each pole.

(1) point z = 0
                −1           −1            − z ( −3)( −4) − z 2 ( −3)( −4)( −5) − z 3
 f (z) =                 3
                           =    [1 + ( −3)( ) +          ( ) +                 ( ) + ...]
         8 z (1 − z / 2)     8z             2      2!      2           3!        2
        1    3  3 5z 2
      =− − − z−        − ...             z = 0 is a pole of order 1
        8 z 16 16 32
(2) point z = 2 ⇒ set z − 2 = ξ ⇒ z( z − 2) 3 = ( 2 + ξ )ξ 3 = 2ξ 3 (1 + ξ / 2)
              1           1       ξ      ξ        ξ        ξ
 f (z) = 3            = 3 [1 − ( ) + ( )2 − ( )3 + ( )4 − ...]
        2ξ (1 + ξ / 2) 2ξ         2      2         2       2
          1       1     1    1 ξ                 1             1            1      1 z−2
      =       −       +   − +        − .. =             −             +          − +     −
         2ξ 3
                4ξ 2   8ξ 16 32             2( z − 2) 3
                                                          4( z − 2) 2   8( z − 2) 16  32
z = 2 is a pole of order 3, the residue of f ( z ) at z = 2 is 1 / 8.
How to obtain the residue ?
              a− m                        a−1
f (z) =                   + ...... +              + a0 + a1 ( z − z0 ) + a 2 ( z − z0 )2 + ...
           ( z − z0 ) m                ( z − z0 )
⇒ ( z − z0 )m f ( z ) = a − m + a − m +1 ( z − z0 ) + ....... + a −1 ( z − z0 )m −1 + ...
    d m −1                                                   ∞
⇒        m −1
              [( z − z0 ) f ( z )] = ( m − 1)! a −1 + ∑ bn ( z − z0 )n
                          m
    dz                                                      n =1
Take the limit z → z0
                              1     d m −1
R( z0 ) = a −1   = lim {                    [( z − z0 ) m f ( z )]} residue at z = z0
                   z → z0 ( m − 1)! dz m −1

(1) For a simple pole m = 1 ⇒ R( z0 ) = lim [( z − z0 ) f ( z )]
                                                         z → z0
                                                                   g( z )
(2) If f ( z ) has a simple at z = z0 and f ( z ) =                       , g ( z ) is analytic and
                                                                   h( z )
    non - zero at z0 and h( z0 ) = 0
                       ( z − z0 ) g ( z )                  ( z − z0 )                     1       g( z )
⇒ R( z0 ) = lim                           = g ( z0 ) lim              = g ( z0 ) lim '          = ' 0
                z → z0      h( z )                   z → z0 h( z )               z → z0 h ( z )  h ( z0 )
Ex : Suppose that f ( z ) has a pole of order m at the point z = z0 . By
 considering the Laurent series of f ( z ) about z0 , deriving a general
 expression for the residue R( z0 ) of f ( z ) at z = z0 . Hence evaluate
                                                 exp iz
 the residue of the function f ( z ) =            2       2
                                                              at the point z = i .
                                               ( z + 1)

           exp iz              exp iz
f (z) =     2       2
                        =        2        2
                                              poles of order 2 at z = i and z = − i
         ( z + 1)          (z + i) (z − i)
for pole at z = i :
 d                          d exp iz                i                 2
   [( z − i ) 2 f ( z )] = [             2
                                           ]=           2
                                                          exp iz −         3
                                                                             exp iz
dz                         dz ( z + i )         (z + i)            (z + i)
         1       i               2              −i
R( i ) = [             e −1 −          e −1 ] =
         1! ( 2i ) 2          ( 2i ) 3          2e
20.14 Residue theorem
f ( z ) has a pole of order m at z = z0                                                C
                        ∞                                                                  γ
          f (z) =       ∑ a n ( z − z0 )      n

                    n= − m                                                             ρ •     z0
I = ∫ f ( z )dz = ∫ f ( z )dz
      C                     γ

set z = z0 + ρe iθ ⇒ dz = iρe iθ dθ
       ∞                                          ∞        2π
I=    ∑ an ∫ ( z − z0 ) dz =
                C
                                      n
                                                ∑ an ∫    0
                                                                iρ n+1e i ( n+1)θ dθ
     n= − m                                   n= − m
                        2π           n +1 i ( n+1)θ        iρ n+1e i ( n+1)θ 2π
for n ≠ −1 ⇒ ∫                  iρ        e           dθ =                   |0 = 0
                        0                                      i ( n + 1)
                    2π
for n = 1 ⇒ ∫               idθ = 2πi
                    0

I = ∫ f ( z )dz = 2πia −1
      C
Residue theorem:
 f ( z ) is continuous within and on a closed contour C
and analytic, except for a finite number of poles within C

                 ∫C f ( z )dz = 2πi ∑ R j
                                    j

∑ R j is the sum of the residues of     f ( z ) at its poles within C
 j




                     C                              C'
The integral I of f ( z ) along an open contour C
                                                                                        C

if f ( z ) has a simple pole at z = z0                                                      ρ
⇒ f ( z ) = φ ( z ) + a−1 ( z − z0 ) −1                                                     z0
φ ( z ) is analytic within some neighbour surrounding z0
| z − z0 |= ρ and θ1 ≤ arg( z − z0 ) ≤ θ 2
ρ is chosen small enough that no singularity of f ( z ) except z = z0
I = ∫ f ( z )dz = ∫ φ ( z )dz + a −1 ∫ ( z − z0 ) −1 dz
      C               C                        C

lim
ρ →0 C
      ∫   φ ( z )dz = 0

                                          θ2       1
I = lim    ∫   f ( z )dz = lim (a −1 ∫             iθ
                                                      iρe iθ dθ ) = ia −1 (θ 2 − θ1 )
      ρ →0 C               ρ →0  ρe       θ1

for a closed contour θ 2 = θ1 + 2π ⇒ I = 2πia −1
20.16 Integrals of sinusoidal functions
       2π
      ∫0    F (cos θ , sin θ )dθ set z = exp iθ in unit circle
                   1      1          1     1
      ⇒ cos θ =      ( z + ), sin θ = ( z − ), dθ = − iz −1dz
                   2      z          2i    z
                       2π           cos 2θ
Ex : Evaluate I = ∫          2      2
                                                dθ for b > a > 0
                       0    a + b − 2ab cos θ

           1 n                        1
cos nθ =     ( z + z − n ) ⇒ cos 2θ = ( z 2 + z − 2 )
           2                          2
                               1 2                                     1
       cos 2θ                    ( z + z − 2 )( − iz −1 )dz         − ( z 4 + 1)idz
                       dθ = 2                                = 2       2
a 2 + b 2 − 2ab cos θ         2     2           1         −1  z ( za 2 + zb 2 − abz 2 − ab )
                             a + b − 2ab ⋅ ( z + z )
                                                2
                               i          ( z 4 + 1)dz        i      ( z 4 + 1)
                            =                              =                      dz
                              2ab    2 2        a      b     2ab 2       a      b
                                    z ( z − z ( − + ) + 1)      z ( z − )( z − )
                                                b     a                  b      a
i                z4 + 1
   2ab ∫C
I=                             dz double poles at z = 0 and z = a / b within the unit circle
                      a      b
             z 2 ( z − )( z − )
                      b      a
                                1     d m −1
Residue : R( z0 ) = lim {                     [( z − z0 ) m f ( z )]
                     z → z0 ( m − 1)! dz m −1

(1) pole at z = 0, m = 2
             1 d 2            z4 + 1
R(0) = lim {      [z 2                            ]}
       z →0 1! dz    z ( z − a / b )( z − b / a )
                       4z 3               ( z 4 + 1)( −1)[2 z − (a / b + b / a )]
      = lim {                           +                                         }= a/b+b/a
        z →0 ( z − a / b )( z − b / a )                      2
                                                 ( z − a / b) ( z − b / a ) 2

(2) pole at z = a / b, m = 1
                                          z4 + 1                         (a / b)4 + 1              − (a 4 + b 4 )
R(a / b) = lim [( z − a / b )     2
                                                               ]=          2
                                                                                               =
            z →a / b            z ( z − a / b )( z − b / a )        (a / b) (a / b − b / a )       ab(b 2 − a 2 )
           i a 2 + b2     a 4 + b4       2πa 2
I = 2πi ×     [       −              ]= 2 2
          2ab   ab      ab(b − a ) b (b − a 2 )
                             2     2
20.17 Some infinite integrals
                            ∞
                           ∫−∞ f ( x )dx
 f ( z ) has the following properties :
(1) f ( z ) is analytic in the upper half - plane, Im z ≥ 0, except for
     a finite number of poles, none of which is on the real axis.
(2) on a semicircle Γ of radius R, R times the maximum of                    y
      | f | on Γ tends to zero as R → ∞ (a sufficient condition
      is that zf ( z ) → 0 as | z |→ ∞ ).                                        Γ
        0                        ∞
(3) ∫           f ( x )dx and   ∫0   f ( x )dx both exist
    −∞                                                                                   x
                ∞
   ⇒        ∫− ∞ f ( x )dx = 2π i ∑ R j                            −R        0       R
                                     j

for | ∫ f ( z )dz |≤ 2π R × (maximum of | f | on Γ ), the integral along Γ
            Γ
tends to zero as R → ∞ .
∞            dx
      Ex : Evaluate I =          ∫0        2
                                       (x + a )     2 4
                                                          a is real

          dz               R          dx                      dz
∫C ( z 2 + a 2 )4     =   ∫− R ( x 2 + a 2 )4 ∫Γ ( z 2 + a 2 )4
                                                +                      as R → ∞
                                                                                                  Γ
                dz                              dz             ∞        dx                   ai
⇒∫                        →0⇒         ∫C ( z 2 + a 2 )4   =   ∫− ∞ ( x 2 + a 2 )4
     Γ   ( z 2 + a 2 )4
( z 2 + a 2 )4 = 0 ⇒ poles of order 4 at z = ± ai ,                                 −R   0            R
only z = ai at the upper half - plane
                                    1               1            1           iξ − 4
set z = ai + ξ , ξ → 0 ⇒ 2            2 4
                                           =           2 4
                                                           =          4
                                                                        (1 −    )
                             (z + a )        ( 2aiξ + ξ )    ( 2aiξ )        2a
                              1 ( −4)( −5)( −6) − i 3      − 5i
the coefficient of ξ −1 is                        ( ) =
                           ( 2a ) 4       3!        2a     32a 7
 ∞         dx                   − 5i           10π                 1 10π      5π
∫0   ( x 2 + a 2 )4
                      = 2πi (
                                32a   7
                                        )=
                                               32a 7
                                                       ⇒I =         ×
                                                                   2 32a 7
                                                                           =
                                                                             32a 7
For poles on the real axis:                                                                     y
                                                                                                            Γ
Principal value of the integral, defined as ρ → 0
     R                        z0 − ρ                   R
P∫        f ( x )dx =     ∫− R         f ( x )dx +    ∫z + ρ   f ( x )dx                              γ ρ
     −R                                                 0
                                                                                    -R            0      z0     R   x
for a closed contour C
                    z0 − ρ                                             R
∫C   f ( z )dz =   ∫− R         f ( x )dx +   ∫γ     f ( z )dz +   ∫z + ρ
                                                                       0
                                                                            f ( x )dx +   ∫Γ f ( z )dz
                          R
                = P∫            f ( x )dx +    ∫γ    f ( z )dz +   ∫Γ f ( z )dz
                        −R

(1) for ∫ f ( z )dz has a pole at z = z0 ⇒ ∫ f ( z )dz = −πia1
           γ                                                       γ
                                             iθ
(2) for ∫ f ( z )dz set z = Re                      dz = i Re iθ dθ
           Γ

     ⇒    ∫Γ   f ( z )dz =      ∫Γ     f (Re iθ )i Re iθ dθ

If f ( z ) vanishes faster than 1 / R 2 as R → ∞, the integral is zero
Jordan’s lemma
(1) f ( z ) is analytic in the upper half - plane except for a finite
    number of poles in Im z > 0
( 2) the maximum of | f ( z ) |→ 0 as | z |→ ∞ in the upper half - plane
(3) m > 0, then

              ∫Γ e
                     imz
    IΓ =                   f ( z )dz → 0 as R → ∞, Γ is the semicircular contour

for 0 ≤ θ ≤ π / 2, 1 ≥ sin θ / θ ≥ π / 2                            f (θ )         f = 2θ / π
| exp(imz )| = | exp(− mR sin θ )|
                                       π
IΓ ≤   ∫Γ   |e imz f ( z )||dz| ≤ MR ∫ e − mR sin θ dθ
                                       0
                                                                                     f = sin θ
                                               π/ 2 − mR sin θ
                                  = 2 MR ∫         e           dθ
                                               0
M is the maximum of |f ( z )| on |z| = R, R → ∞ M → 0                        π /2           θ
                          πM
                π / 2 − mR ( 2θ / π )        πM
I Γ < 2 MR ∫         e       (1 − e − mR ) <
                                      dθ   =
                0          m                  m
as R → ∞ ⇒ M → 0 ⇒ I Γ → 0
cos mx
                                              ∞
Ex : Find the principal value of          ∫− ∞ x − a dx a real, m > 0                             Γ
                                       e imz
Consider the integral I =         ∫C   z−a
                                             dz = 0 no pole in the
                                                                                            γ ρ
                                         −1
upper half - plane, and |( z − a ) | → 0 as |z| → ∞
                                                                              -R        0     a       R
           e imz
I =   ∫C   z−a
                 dz

       a−ρ    e imx             e imz             R    e imx             e imz
  =   ∫− R    x−a
                    dx +   ∫γ   z−a
                                      dz +    ∫a + ρ   x−a
                                                             dx +   ∫Γ   z−a
                                                                               dz = 0

                                       e imz
As R → ∞ and ρ → 0 ⇒ ∫                       dz → 0
                                   Γ   z−a
       ∞e imx
⇒ P∫          dx − iπa−1 = 0 and a−1 = e ima
    −∞ x − a
     ∞ cos mx                         ∞ sin mx
⇒ P∫           dx = −π sin ma and P ∫          dx = π cos ma
    −∞ x − a                         −∞ x − a
20.18 Integral of multivalued functions
                                          1/ 2
                                                                              y
Multivalued functions such as z , Lnz
Single branch point is at the otigin. We let R → ∞                            R       Γ
and ρ → 0. The integrand is multivalued, its values
                                                                          γ
along two lines AB and CD joining z = ρ to z = R                                  A   B
are not equal and opposite.                                               ρ               x
                                                                                  C   D
              ∞          dx
 Ex : I =    ∫0   ( x + a )3 x1 / 2
                                      for a > 0


(1) the integrand f ( z ) = ( z + a ) −3 z −1 / 2 , |zf ( z )| → 0 as ρ → 0 and R → ∞
     the two circles make no contribution to the contour integral
(2) pole at z = − a , and ( − a )1 / 2 = a 1 / 2 e iπ / 2 = ia 1 / 2
                            1     d 3 −1                     1
     R( − a ) = lim                        [( z + a )3                ]
                z → − a ( 3 − 1)! dz 3 − 1                     3 1/ 2
                                                       (z + a) z
                        1 d 2 −1 / 2    − 3i
              = lim             z    =
                z → − a 2! dz 2        8a 5 / 2
− 3i
∫AB   dz +   ∫Γ   dz +   ∫DC   dz +   ∫γ   dz = 2πi (
                                                        8a   5/2
                                                                   )

and ∫ dz = 0 and           ∫γ dz = 0
       Γ

along line AB ⇒ z = xe i 0 , along line CD ⇒ z = xe i 2π
 ∞              dx             0                   dx                     3π
∫0, A→ B ( x + a )3 x1 / 2 ∞,C → D ( xe i 2π + a )3 x1 / 2e (1 / 2×2πi ) 4a 5 / 2
                            +∫                                          =

           1 ∞            dx            3π
⇒ (1 − iπ )∫                         =
         e     0 ( x + a )3 x1 / 2     4a 5 / 2
     ∞        dx             3π
⇒∫                        =
     0 ( x + a )3 x1 / 2    8a 5 / 2
∞     x sin x
    Ex : Evaluate I (σ ) =                    ∫− ∞ x 2 − σ 2 dx
         z sin z      1                         ze iz       1                  ze − iz
   ∫C   z2 − σ 2
                 dz =
                      2i             ∫C   1   z2 − σ 2
                                                       dz −
                                                            2i        ∫C   2
                                                                               2
                                                                               z −σ      2
                                                                                             dz = I1 + I 2

(1) for I1 , the contour is choosed on the upper half - plane                                                     C1
   due to the term e iz , and only one pole at z = σ .                                                       Γ
     1               ze iz       1                 −σ − ρ     xe ix                                 γ1
I1 =
     2i   ∫C   1   z2 − σ 2
                            dz =
                                 2i             ∫− R         x2 − σ   2
                                                                        dx
                                                                                                      ρ
     1     σ −ρ              xe ix   1                   ∞      xe ix                          -R     -σ         σ     R
   +
     2i   ∫−σ + ρ      x2 − σ 2
                                dx +
                                     2i                 ∫σ + ρ x 2 − σ 2 dx                                      γ2

     1               ze iz      1        ze iz      1                                    ze iz
   +
     2i   ∫γ   1   z2 − σ 2
                            dz + ∫ 2
                                2i γ 2 z − σ 2
                                               dz +
                                                    2i                          ∫Γ z 2 − σ 2 dz
     1                         σe iσ  π
   =    2πi × Res( z = σ ) = π       = e iσ
     2i                         2σ    2
As ρ → 0 and R → ∞ ⇒ ∫ dz → 0
                                            Γ

1                    ze iz           1                           − π − iσ
2i   ∫γ   1   ( z + σ )( z − σ )
                                dz =
                                     2i
                                        × ( −πi )Res( z = −σ ) =
                                                                  4
                                                                    e

1                    ze iz           1                    π
     ∫γ                         dz =    × πiRes( z = σ ) = e iσ
2i        2   ( z + σ )( z − σ )     2i                   4
       1 ∞        xe ix       π                 π                                                 γ1
                         dx + (e iσ − e − iσ ) = e iσ
      2i ∫− ∞ x 2 − σ 2
I1 =                                                                                                       σ
                              4                 2
                                                                                                  −σ
(2) for I 2 , we choose the lower half - plane by the
                     − iz                                                                    Γ             γ2
     term e                 , only one pole at z = −σ
     −1       ze − iz      − 1 −σ − ρ xe − ix
I2 =
     2i ∫C2 z 2 − σ 2 dz = 2i ∫− R x 2 − σ 2 dx                                                                C2

       1         σ −ρ            xe − ix    1      ∞         xe − ix      1             ze − iz
     −
       2i       ∫−σ + ρ       x2 − σ 2
                                       dx −
                                            2i    ∫σ + ρ    x2 − σ 2
                                                                     dx −
                                                                          2i   ∫γ   1
                                                                                        2
                                                                                        z −σ      2
                                                                                                      dz

       1                    ze − iz    1          ze − iz        −1              ( −σ )e iσ  π
     −          ∫γ                dz −      ∫Γ            dz = (    ) × ( −2πi )            = e iσ
       2i            2   z2 − σ 2      2i        z2 − σ 2        2i                 − 2σ     2
As ρ → 0, R → ∞ ⇒ ∫ dz → 0
                          Γ

−1            ze − iz            −1          ( −σ )e iσ  π
                                                        = e iσ
2i ∫γ 1 ( z + σ )( z − σ )
                          dz = (    )( −πi )
                                 2i             − 2σ     4
−1            ze − iz            −1        σe − iσ   − π − iσ
2i ∫γ 2 ( z + σ )( z − σ )dz = (
                                 2i
                                    )(πi )
                                            2σ
                                                   =
                                                      4
                                                        e

     − 1 ∞ xe − ix         π                  π
                       dx + (e iσ − e − iσ ) = e iσ
     2i ∫− ∞ x 2 − σ 2
I2 =
                           4                  2
  − 1 ∞ xe − ix         π      1
                    dx = e iσ − (e iσ − e − iσ )
  2i ∫− ∞ x 2 − σ 2
⇒
                        2      4
         ∞    x sin x        1 ∞        xe ix       1 ∞ xe − ix
I (σ ) = ∫
                             2i ∫− ∞ x 2 − σ 2      2i ∫− ∞ x 2 − σ 2
                       dx =                    dx −                   dx
          −∞ x 2 − σ 2

          π        π                   π        π
       = e iσ − ( e iσ − e − iσ ) + e iσ − ( e iσ − e − iσ )
          2         4                  2         4
                 π        π            π
       = πe iσ − e iσ + e − iσ = (e iσ + e − iσ ) = π cos σ
                  2        2           2

More Related Content

What's hot

Leaner algebra presentation (ring)
Leaner algebra presentation (ring)Leaner algebra presentation (ring)
Leaner algebra presentation (ring)Muhammad Umar Farooq
 
Laurent's Series & Types of Singularities
Laurent's Series & Types of SingularitiesLaurent's Series & Types of Singularities
Laurent's Series & Types of SingularitiesAakash Singh
 
Vector calculus
Vector calculusVector calculus
Vector calculusraghu ram
 
30 surface integrals
30 surface integrals30 surface integrals
30 surface integralsmath267
 
Continutiy of Functions.ppt
Continutiy of Functions.pptContinutiy of Functions.ppt
Continutiy of Functions.pptLadallaRajKumar
 
Linear dependence & independence vectors
Linear dependence & independence vectorsLinear dependence & independence vectors
Linear dependence & independence vectorsRakib Hossain
 
complex variable PPT ( SEM 2 / CH -2 / GTU)
complex variable PPT ( SEM 2 / CH -2 / GTU)complex variable PPT ( SEM 2 / CH -2 / GTU)
complex variable PPT ( SEM 2 / CH -2 / GTU)tejaspatel1997
 
Hausdorff and Non-Hausdorff Spaces
Hausdorff and Non-Hausdorff SpacesHausdorff and Non-Hausdorff Spaces
Hausdorff and Non-Hausdorff Spacesgizemk
 
Fourier series of odd functions with period 2 l
Fourier series of odd functions with period 2 lFourier series of odd functions with period 2 l
Fourier series of odd functions with period 2 lPepa Vidosa Serradilla
 
presentation on Euler and Modified Euler method ,and Fitting of curve
presentation on Euler and Modified Euler method ,and Fitting of curve presentation on Euler and Modified Euler method ,and Fitting of curve
presentation on Euler and Modified Euler method ,and Fitting of curve Mukuldev Khunte
 

What's hot (20)

Leaner algebra presentation (ring)
Leaner algebra presentation (ring)Leaner algebra presentation (ring)
Leaner algebra presentation (ring)
 
Unit1
Unit1Unit1
Unit1
 
Laurent's Series & Types of Singularities
Laurent's Series & Types of SingularitiesLaurent's Series & Types of Singularities
Laurent's Series & Types of Singularities
 
Vector calculus
Vector calculusVector calculus
Vector calculus
 
30 surface integrals
30 surface integrals30 surface integrals
30 surface integrals
 
Complex analysis
Complex analysisComplex analysis
Complex analysis
 
Continutiy of Functions.ppt
Continutiy of Functions.pptContinutiy of Functions.ppt
Continutiy of Functions.ppt
 
Power series
Power seriesPower series
Power series
 
Real analysis
Real analysisReal analysis
Real analysis
 
1523 double integrals
1523 double integrals1523 double integrals
1523 double integrals
 
Complex function
Complex functionComplex function
Complex function
 
Linear dependence & independence vectors
Linear dependence & independence vectorsLinear dependence & independence vectors
Linear dependence & independence vectors
 
complex variable PPT ( SEM 2 / CH -2 / GTU)
complex variable PPT ( SEM 2 / CH -2 / GTU)complex variable PPT ( SEM 2 / CH -2 / GTU)
complex variable PPT ( SEM 2 / CH -2 / GTU)
 
Topology
TopologyTopology
Topology
 
project
projectproject
project
 
Power series
Power series Power series
Power series
 
Hausdorff and Non-Hausdorff Spaces
Hausdorff and Non-Hausdorff SpacesHausdorff and Non-Hausdorff Spaces
Hausdorff and Non-Hausdorff Spaces
 
Lagrange’s interpolation formula
Lagrange’s interpolation formulaLagrange’s interpolation formula
Lagrange’s interpolation formula
 
Fourier series of odd functions with period 2 l
Fourier series of odd functions with period 2 lFourier series of odd functions with period 2 l
Fourier series of odd functions with period 2 l
 
presentation on Euler and Modified Euler method ,and Fitting of curve
presentation on Euler and Modified Euler method ,and Fitting of curve presentation on Euler and Modified Euler method ,and Fitting of curve
presentation on Euler and Modified Euler method ,and Fitting of curve
 

Viewers also liked

Mathematics and History of Complex Variables
Mathematics and History of Complex VariablesMathematics and History of Complex Variables
Mathematics and History of Complex VariablesSolo Hermelin
 
Complex Number I - Presentation
Complex Number I - PresentationComplex Number I - Presentation
Complex Number I - Presentationyhchung
 
Cauchy's integral formula 
Cauchy's integral formula Cauchy's integral formula 
Cauchy's integral formula HanpenRobot
 
Integration in the complex plane
Integration in the complex planeIntegration in the complex plane
Integration in the complex planeAmit Amola
 
Engineering Mathematics-IV_B.Tech_Semester-IV_Unit-I
Engineering Mathematics-IV_B.Tech_Semester-IV_Unit-IEngineering Mathematics-IV_B.Tech_Semester-IV_Unit-I
Engineering Mathematics-IV_B.Tech_Semester-IV_Unit-IRai University
 
Application of analytic function
Application of analytic functionApplication of analytic function
Application of analytic functionDr. Nirav Vyas
 
Cauchy riemann equations
Cauchy riemann equationsCauchy riemann equations
Cauchy riemann equationssajidpk92
 
Complex analysis book by iit
Complex analysis book by iitComplex analysis book by iit
Complex analysis book by iitJITENDRA SUWASIYA
 
Introduction to Mathematical Probability
Introduction to Mathematical ProbabilityIntroduction to Mathematical Probability
Introduction to Mathematical ProbabilitySolo Hermelin
 
Golden words of swami vivekananda
Golden words of swami vivekanandaGolden words of swami vivekananda
Golden words of swami vivekanandachiefs world
 
Complex Analysis - Differentiability and Analyticity (Team 2) - University of...
Complex Analysis - Differentiability and Analyticity (Team 2) - University of...Complex Analysis - Differentiability and Analyticity (Team 2) - University of...
Complex Analysis - Differentiability and Analyticity (Team 2) - University of...Alex Bell
 
Swami Vivekananda Quotes
Swami Vivekananda QuotesSwami Vivekananda Quotes
Swami Vivekananda QuotesMoncy Varghese
 
Cauchy riemann equations
Cauchy riemann equationsCauchy riemann equations
Cauchy riemann equationsHanpenRobot
 
Lecture notes for s4 b tech Mathematics
Lecture notes for s4 b tech Mathematics  Lecture notes for s4 b tech Mathematics
Lecture notes for s4 b tech Mathematics Anoop T Vilakkuvettom
 
Swami vivekananda’s 150 quotes
Swami vivekananda’s 150  quotesSwami vivekananda’s 150  quotes
Swami vivekananda’s 150 quotesFiroj Md Shah
 
Portfolio Presentation 2
Portfolio Presentation 2Portfolio Presentation 2
Portfolio Presentation 2rutheast
 

Viewers also liked (20)

Mathematics and History of Complex Variables
Mathematics and History of Complex VariablesMathematics and History of Complex Variables
Mathematics and History of Complex Variables
 
Complex Number I - Presentation
Complex Number I - PresentationComplex Number I - Presentation
Complex Number I - Presentation
 
Cauchy's integral formula 
Cauchy's integral formula Cauchy's integral formula 
Cauchy's integral formula 
 
Integration in the complex plane
Integration in the complex planeIntegration in the complex plane
Integration in the complex plane
 
Engineering Mathematics-IV_B.Tech_Semester-IV_Unit-I
Engineering Mathematics-IV_B.Tech_Semester-IV_Unit-IEngineering Mathematics-IV_B.Tech_Semester-IV_Unit-I
Engineering Mathematics-IV_B.Tech_Semester-IV_Unit-I
 
Application of analytic function
Application of analytic functionApplication of analytic function
Application of analytic function
 
Cauchy riemann equations
Cauchy riemann equationsCauchy riemann equations
Cauchy riemann equations
 
Matrices i
Matrices iMatrices i
Matrices i
 
Complex analysis book by iit
Complex analysis book by iitComplex analysis book by iit
Complex analysis book by iit
 
Introduction to Mathematical Probability
Introduction to Mathematical ProbabilityIntroduction to Mathematical Probability
Introduction to Mathematical Probability
 
Golden words of swami vivekananda
Golden words of swami vivekanandaGolden words of swami vivekananda
Golden words of swami vivekananda
 
Complex Analysis - Differentiability and Analyticity (Team 2) - University of...
Complex Analysis - Differentiability and Analyticity (Team 2) - University of...Complex Analysis - Differentiability and Analyticity (Team 2) - University of...
Complex Analysis - Differentiability and Analyticity (Team 2) - University of...
 
Swami Vivekananda Quotes
Swami Vivekananda QuotesSwami Vivekananda Quotes
Swami Vivekananda Quotes
 
Cauchy riemann equations
Cauchy riemann equationsCauchy riemann equations
Cauchy riemann equations
 
Lecture notes for s4 b tech Mathematics
Lecture notes for s4 b tech Mathematics  Lecture notes for s4 b tech Mathematics
Lecture notes for s4 b tech Mathematics
 
Vector analysis
Vector analysisVector analysis
Vector analysis
 
Prime numbers
Prime numbersPrime numbers
Prime numbers
 
Swami vivekananda’s 150 quotes
Swami vivekananda’s 150  quotesSwami vivekananda’s 150  quotes
Swami vivekananda’s 150 quotes
 
Complex number
Complex numberComplex number
Complex number
 
Portfolio Presentation 2
Portfolio Presentation 2Portfolio Presentation 2
Portfolio Presentation 2
 

Similar to Complex varible

Beam theory
Beam theoryBeam theory
Beam theorybissla19
 
Calculus First Test 2011/10/20
Calculus First Test 2011/10/20Calculus First Test 2011/10/20
Calculus First Test 2011/10/20Kuan-Lun Wang
 
03 convexfunctions
03 convexfunctions03 convexfunctions
03 convexfunctionsSufyan Sahoo
 
Complex analysis and differential equation
Complex analysis and differential equationComplex analysis and differential equation
Complex analysis and differential equationSpringer
 
Mac331 complex analysis_mfa_week6_16-10-20 (1)
Mac331 complex analysis_mfa_week6_16-10-20 (1)Mac331 complex analysis_mfa_week6_16-10-20 (1)
Mac331 complex analysis_mfa_week6_16-10-20 (1)KINGSHUKMUKHERJEE11
 
Maths assignment
Maths assignmentMaths assignment
Maths assignmentNtshima
 
Introduction to inverse problems
Introduction to inverse problemsIntroduction to inverse problems
Introduction to inverse problemsDelta Pi Systems
 
Lesson20 Tangent Planes Slides+Notes
Lesson20   Tangent Planes Slides+NotesLesson20   Tangent Planes Slides+Notes
Lesson20 Tangent Planes Slides+NotesMatthew Leingang
 
Reflect tsukuba524
Reflect tsukuba524Reflect tsukuba524
Reflect tsukuba524kazuhase2011
 
3.2 Derivative as a Function
3.2 Derivative as a Function3.2 Derivative as a Function
3.2 Derivative as a Functiongregcross22
 
3.2 Derivative as a Function
3.2 Derivative as a Function3.2 Derivative as a Function
3.2 Derivative as a Functiongregcross22
 
Bahan ajar kalkulus integral
Bahan ajar kalkulus integralBahan ajar kalkulus integral
Bahan ajar kalkulus integralgrand_livina_good
 
Week 3 [compatibility mode]
Week 3 [compatibility mode]Week 3 [compatibility mode]
Week 3 [compatibility mode]Hazrul156
 
Basic differential equations in fluid mechanics
Basic differential equations in fluid mechanicsBasic differential equations in fluid mechanics
Basic differential equations in fluid mechanicsTarun Gehlot
 

Similar to Complex varible (20)

U unit3 vm
U unit3 vmU unit3 vm
U unit3 vm
 
Beam theory
Beam theoryBeam theory
Beam theory
 
Calculus First Test 2011/10/20
Calculus First Test 2011/10/20Calculus First Test 2011/10/20
Calculus First Test 2011/10/20
 
Matrix calculus
Matrix calculusMatrix calculus
Matrix calculus
 
03 convexfunctions
03 convexfunctions03 convexfunctions
03 convexfunctions
 
Complex analysis and differential equation
Complex analysis and differential equationComplex analysis and differential equation
Complex analysis and differential equation
 
Mac331 complex analysis_mfa_week6_16-10-20 (1)
Mac331 complex analysis_mfa_week6_16-10-20 (1)Mac331 complex analysis_mfa_week6_16-10-20 (1)
Mac331 complex analysis_mfa_week6_16-10-20 (1)
 
Maths assignment
Maths assignmentMaths assignment
Maths assignment
 
subdiff_prox.pdf
subdiff_prox.pdfsubdiff_prox.pdf
subdiff_prox.pdf
 
Introduction to inverse problems
Introduction to inverse problemsIntroduction to inverse problems
Introduction to inverse problems
 
Lesson20 Tangent Planes Slides+Notes
Lesson20   Tangent Planes Slides+NotesLesson20   Tangent Planes Slides+Notes
Lesson20 Tangent Planes Slides+Notes
 
Reflect tsukuba524
Reflect tsukuba524Reflect tsukuba524
Reflect tsukuba524
 
3.2 Derivative as a Function
3.2 Derivative as a Function3.2 Derivative as a Function
3.2 Derivative as a Function
 
3.2 Derivative as a Function
3.2 Derivative as a Function3.2 Derivative as a Function
3.2 Derivative as a Function
 
Bahan ajar kalkulus integral
Bahan ajar kalkulus integralBahan ajar kalkulus integral
Bahan ajar kalkulus integral
 
Week 3 [compatibility mode]
Week 3 [compatibility mode]Week 3 [compatibility mode]
Week 3 [compatibility mode]
 
Basic differential equations in fluid mechanics
Basic differential equations in fluid mechanicsBasic differential equations in fluid mechanics
Basic differential equations in fluid mechanics
 
lec12.pdf
lec12.pdflec12.pdf
lec12.pdf
 
Double integration
Double integrationDouble integration
Double integration
 
Sect1 2
Sect1 2Sect1 2
Sect1 2
 

More from Naveen Sihag

More from Naveen Sihag (20)

A P J Abdul Kalam
A P J Abdul KalamA P J Abdul Kalam
A P J Abdul Kalam
 
Rise to power adolf hitler
Rise to power adolf hitlerRise to power adolf hitler
Rise to power adolf hitler
 
Networking
NetworkingNetworking
Networking
 
Efective computing
Efective computingEfective computing
Efective computing
 
Bluetooth 1
Bluetooth 1Bluetooth 1
Bluetooth 1
 
Black holes
Black holesBlack holes
Black holes
 
Bluetooth 1
Bluetooth 1Bluetooth 1
Bluetooth 1
 
Black holes
Black holesBlack holes
Black holes
 
Visible light communication
Visible light communicationVisible light communication
Visible light communication
 
Variable frequency drives
Variable frequency drivesVariable frequency drives
Variable frequency drives
 
Usb
UsbUsb
Usb
 
Transducers
TransducersTransducers
Transducers
 
Touch screen technology
Touch screen technologyTouch screen technology
Touch screen technology
 
Solids and semiconductors
Solids and semiconductorsSolids and semiconductors
Solids and semiconductors
 
Sms &mms
Sms &mmsSms &mms
Sms &mms
 
Robotics and collision detection
Robotics and   collision detectionRobotics and   collision detection
Robotics and collision detection
 
Renewable energy
Renewable energyRenewable energy
Renewable energy
 
Red tacton
Red tactonRed tacton
Red tacton
 
Pulse code modulation
Pulse code modulationPulse code modulation
Pulse code modulation
 
Paper battery
Paper batteryPaper battery
Paper battery
 

Recently uploaded

Mastering the Unannounced Regulatory Inspection
Mastering the Unannounced Regulatory InspectionMastering the Unannounced Regulatory Inspection
Mastering the Unannounced Regulatory InspectionSafetyChain Software
 
Employee wellbeing at the workplace.pptx
Employee wellbeing at the workplace.pptxEmployee wellbeing at the workplace.pptx
Employee wellbeing at the workplace.pptxNirmalaLoungPoorunde1
 
Industrial Policy - 1948, 1956, 1973, 1977, 1980, 1991
Industrial Policy - 1948, 1956, 1973, 1977, 1980, 1991Industrial Policy - 1948, 1956, 1973, 1977, 1980, 1991
Industrial Policy - 1948, 1956, 1973, 1977, 1980, 1991RKavithamani
 
Nutritional Needs Presentation - HLTH 104
Nutritional Needs Presentation - HLTH 104Nutritional Needs Presentation - HLTH 104
Nutritional Needs Presentation - HLTH 104misteraugie
 
How to Make a Pirate ship Primary Education.pptx
How to Make a Pirate ship Primary Education.pptxHow to Make a Pirate ship Primary Education.pptx
How to Make a Pirate ship Primary Education.pptxmanuelaromero2013
 
A Critique of the Proposed National Education Policy Reform
A Critique of the Proposed National Education Policy ReformA Critique of the Proposed National Education Policy Reform
A Critique of the Proposed National Education Policy ReformChameera Dedduwage
 
Call Girls in Dwarka Mor Delhi Contact Us 9654467111
Call Girls in Dwarka Mor Delhi Contact Us 9654467111Call Girls in Dwarka Mor Delhi Contact Us 9654467111
Call Girls in Dwarka Mor Delhi Contact Us 9654467111Sapana Sha
 
Kisan Call Centre - To harness potential of ICT in Agriculture by answer farm...
Kisan Call Centre - To harness potential of ICT in Agriculture by answer farm...Kisan Call Centre - To harness potential of ICT in Agriculture by answer farm...
Kisan Call Centre - To harness potential of ICT in Agriculture by answer farm...Krashi Coaching
 
Q4-W6-Restating Informational Text Grade 3
Q4-W6-Restating Informational Text Grade 3Q4-W6-Restating Informational Text Grade 3
Q4-W6-Restating Informational Text Grade 3JemimahLaneBuaron
 
Introduction to ArtificiaI Intelligence in Higher Education
Introduction to ArtificiaI Intelligence in Higher EducationIntroduction to ArtificiaI Intelligence in Higher Education
Introduction to ArtificiaI Intelligence in Higher Educationpboyjonauth
 
Hybridoma Technology ( Production , Purification , and Application )
Hybridoma Technology  ( Production , Purification , and Application  ) Hybridoma Technology  ( Production , Purification , and Application  )
Hybridoma Technology ( Production , Purification , and Application ) Sakshi Ghasle
 
Activity 01 - Artificial Culture (1).pdf
Activity 01 - Artificial Culture (1).pdfActivity 01 - Artificial Culture (1).pdf
Activity 01 - Artificial Culture (1).pdfciinovamais
 
Interactive Powerpoint_How to Master effective communication
Interactive Powerpoint_How to Master effective communicationInteractive Powerpoint_How to Master effective communication
Interactive Powerpoint_How to Master effective communicationnomboosow
 
Arihant handbook biology for class 11 .pdf
Arihant handbook biology for class 11 .pdfArihant handbook biology for class 11 .pdf
Arihant handbook biology for class 11 .pdfchloefrazer622
 
URLs and Routing in the Odoo 17 Website App
URLs and Routing in the Odoo 17 Website AppURLs and Routing in the Odoo 17 Website App
URLs and Routing in the Odoo 17 Website AppCeline George
 
Z Score,T Score, Percential Rank and Box Plot Graph
Z Score,T Score, Percential Rank and Box Plot GraphZ Score,T Score, Percential Rank and Box Plot Graph
Z Score,T Score, Percential Rank and Box Plot GraphThiyagu K
 
Sanyam Choudhary Chemistry practical.pdf
Sanyam Choudhary Chemistry practical.pdfSanyam Choudhary Chemistry practical.pdf
Sanyam Choudhary Chemistry practical.pdfsanyamsingh5019
 
The basics of sentences session 2pptx copy.pptx
The basics of sentences session 2pptx copy.pptxThe basics of sentences session 2pptx copy.pptx
The basics of sentences session 2pptx copy.pptxheathfieldcps1
 
Web & Social Media Analytics Previous Year Question Paper.pdf
Web & Social Media Analytics Previous Year Question Paper.pdfWeb & Social Media Analytics Previous Year Question Paper.pdf
Web & Social Media Analytics Previous Year Question Paper.pdfJayanti Pande
 

Recently uploaded (20)

Mastering the Unannounced Regulatory Inspection
Mastering the Unannounced Regulatory InspectionMastering the Unannounced Regulatory Inspection
Mastering the Unannounced Regulatory Inspection
 
Employee wellbeing at the workplace.pptx
Employee wellbeing at the workplace.pptxEmployee wellbeing at the workplace.pptx
Employee wellbeing at the workplace.pptx
 
Industrial Policy - 1948, 1956, 1973, 1977, 1980, 1991
Industrial Policy - 1948, 1956, 1973, 1977, 1980, 1991Industrial Policy - 1948, 1956, 1973, 1977, 1980, 1991
Industrial Policy - 1948, 1956, 1973, 1977, 1980, 1991
 
Nutritional Needs Presentation - HLTH 104
Nutritional Needs Presentation - HLTH 104Nutritional Needs Presentation - HLTH 104
Nutritional Needs Presentation - HLTH 104
 
How to Make a Pirate ship Primary Education.pptx
How to Make a Pirate ship Primary Education.pptxHow to Make a Pirate ship Primary Education.pptx
How to Make a Pirate ship Primary Education.pptx
 
Mattingly "AI & Prompt Design: The Basics of Prompt Design"
Mattingly "AI & Prompt Design: The Basics of Prompt Design"Mattingly "AI & Prompt Design: The Basics of Prompt Design"
Mattingly "AI & Prompt Design: The Basics of Prompt Design"
 
A Critique of the Proposed National Education Policy Reform
A Critique of the Proposed National Education Policy ReformA Critique of the Proposed National Education Policy Reform
A Critique of the Proposed National Education Policy Reform
 
Call Girls in Dwarka Mor Delhi Contact Us 9654467111
Call Girls in Dwarka Mor Delhi Contact Us 9654467111Call Girls in Dwarka Mor Delhi Contact Us 9654467111
Call Girls in Dwarka Mor Delhi Contact Us 9654467111
 
Kisan Call Centre - To harness potential of ICT in Agriculture by answer farm...
Kisan Call Centre - To harness potential of ICT in Agriculture by answer farm...Kisan Call Centre - To harness potential of ICT in Agriculture by answer farm...
Kisan Call Centre - To harness potential of ICT in Agriculture by answer farm...
 
Q4-W6-Restating Informational Text Grade 3
Q4-W6-Restating Informational Text Grade 3Q4-W6-Restating Informational Text Grade 3
Q4-W6-Restating Informational Text Grade 3
 
Introduction to ArtificiaI Intelligence in Higher Education
Introduction to ArtificiaI Intelligence in Higher EducationIntroduction to ArtificiaI Intelligence in Higher Education
Introduction to ArtificiaI Intelligence in Higher Education
 
Hybridoma Technology ( Production , Purification , and Application )
Hybridoma Technology  ( Production , Purification , and Application  ) Hybridoma Technology  ( Production , Purification , and Application  )
Hybridoma Technology ( Production , Purification , and Application )
 
Activity 01 - Artificial Culture (1).pdf
Activity 01 - Artificial Culture (1).pdfActivity 01 - Artificial Culture (1).pdf
Activity 01 - Artificial Culture (1).pdf
 
Interactive Powerpoint_How to Master effective communication
Interactive Powerpoint_How to Master effective communicationInteractive Powerpoint_How to Master effective communication
Interactive Powerpoint_How to Master effective communication
 
Arihant handbook biology for class 11 .pdf
Arihant handbook biology for class 11 .pdfArihant handbook biology for class 11 .pdf
Arihant handbook biology for class 11 .pdf
 
URLs and Routing in the Odoo 17 Website App
URLs and Routing in the Odoo 17 Website AppURLs and Routing in the Odoo 17 Website App
URLs and Routing in the Odoo 17 Website App
 
Z Score,T Score, Percential Rank and Box Plot Graph
Z Score,T Score, Percential Rank and Box Plot GraphZ Score,T Score, Percential Rank and Box Plot Graph
Z Score,T Score, Percential Rank and Box Plot Graph
 
Sanyam Choudhary Chemistry practical.pdf
Sanyam Choudhary Chemistry practical.pdfSanyam Choudhary Chemistry practical.pdf
Sanyam Choudhary Chemistry practical.pdf
 
The basics of sentences session 2pptx copy.pptx
The basics of sentences session 2pptx copy.pptxThe basics of sentences session 2pptx copy.pptx
The basics of sentences session 2pptx copy.pptx
 
Web & Social Media Analytics Previous Year Question Paper.pdf
Web & Social Media Analytics Previous Year Question Paper.pdfWeb & Social Media Analytics Previous Year Question Paper.pdf
Web & Social Media Analytics Previous Year Question Paper.pdf
 

Complex varible

  • 1. f ( z ) = u( x , y ) + iv ( x , y ) for z = x + iy f ( z + ∆z ) − f ( z ) f ' ( z ) = lim [ ] exists ∆z →0 ∆z Its value does not depend on the direction. Ex : Show that the function f ( z ) = x 2 − y 2 + i 2 xy is differentiable for all values of z . for ∆z = ∆x + i∆y f ( z + ∆z ) − f ( z ) f ' ( z ) = lim ∆z →0 ∆z ( x + ∆x ) 2 − ( y + ∆y ) 2 + 2i ( x + ∆x )( y + ∆y ) − x 2 + y 2 − 2ixy = ∆x + i∆y ( ∆x ) 2 − ( ∆y )2 + 2i∆x∆y = 2x + i2 y + ∆x + i∆y (1) choose ∆y = 0, ∆x → 0 ⇒ f ' ( z ) = 2 x + i 2 y (2) choose ∆x = 0, ∆y → 0 ⇒ f ' ( z ) = 2 x + i 2 y
  • 2. * * Another method : f ( z ) = ( x + iy ) 2 = z 2 ' ( z + ∆z )2 − z 2 ( ∆z )2 + 2 z∆z f ( z ) = lim [ ] = lim [ ] ∆z →0 ∆z ∆z → 0 ∆z = lim ∆z + 2 z = 2 z ∆z → 0 Ex : Show that the function f ( z ) = 2 y + ix is not differentiable anywhere in the complex plane. f ( z + ∆z ) − f ( z ) 2 y + 2∆y + ix + i∆x − 2 y − ix 2∆y + i∆x = = ∆z ∆x + i∆y ∆x + i∆y if ∆z → 0 along a line thriugh z of slope m ⇒ ∆y = m ∆x f ( z + ∆z ) − f ( z ) 2 ∆ y + i∆ x 2m + i f ' ( z ) = lim = lim [ ]= ∆z →0 ∆z ∆x ,∆y →0 ∆x + i∆y 1 + im The limit depends on m (the direction), so f ( z ) is nowhere differentiable.
  • 3. Ex : Show that the function f ( z ) = 1 /(1 − z ) is analytic everywhere except at z = 1. f ( z + ∆z ) − f ( z ) 1 1 1 f ' ( z ) = lim [ ] = lim [ ( − )] ∆z →0 ∆z ∆z →0 ∆z 1 − z − ∆z 1 − z 1 1 = lim [ ]= ∆z →0 (1 − z − ∆z )(1 − z ) (1 − z ) 2 Provided z ≠ 1, f ( z ) is analytic everywhere such that f ' ( z ) is independent of the direction.
  • 4. 20.2 Cauchy-Riemann relation A function f(z)=u(x,y)+iv(x,y) is differentiable and analytic, there must be particular connection between u(x,y) and v(x,y) f ( z + ∆z ) − f ( z ) L = lim [ ] ∆z →0 ∆z f ( z ) = u( x , y ) + iv ( x , y ) ∆z = ∆x + i∆y f ( z + ∆z ) = u( x + ∆x , y + ∆y ) + iv ( x + ∆x , y + ∆y ) u( x + ∆x , y + ∆y ) + iv ( x + ∆x , y + ∆y ) − u( x , y ) − iv ( x , y ) ⇒ L = lim [ ] ∆x ,∆y →0 ∆x + i∆y (1) if suppose ∆z is real ⇒ ∆y = 0 u( x + ∆x , y ) − u( x , y ) v ( x + ∆x , y ) − v ( x , y ) ∂u ∂v ⇒ L = lim [ +i ]= +i ∆x →0 ∆x ∆x ∂x ∂x (2) if suppose ∆z is imaginary ⇒ ∆x = 0 u( x , y + ∆y ) − u( x , y ) v ( x , y + ∆y ) − v ( x , y ) ∂u ∂v ⇒ L = lim [ +i ] = −i + ∆y →0 i ∆y i ∆y ∂y ∂y ∂u ∂v ∂v ∂u = and =- Cauchy - Riemann relations ∂x ∂y ∂x ∂y
  • 5. Ex : In which domain of the complex plane is f ( z ) =| x | − i | y | an analytic function? u( x , y ) =| x |, v ( x , y ) = − | y | ∂u ∂v ∂ ∂ (1) = ⇒ | x |= [− | y |] ⇒ (a) x > 0, y < 0 the fouth quatrant ∂x ∂y ∂x ∂y (b) x < 0, y > 0 the second quatrant ∂v ∂u ∂ ∂ (2) =− ⇒ [− | y |] = − | x | ∂x ∂y ∂x ∂y z = x + iy and complex conjugate of z is z * = x − iy ⇒ x = ( z + z * ) / 2 and y = ( z − z * ) / 2i ∂f ∂f ∂x ∂f ∂y 1 ∂u ∂v i ∂v ∂u ⇒ = + = ( − )+ ( + ) ∂z * ∂x ∂z * ∂y ∂z * 2 ∂x ∂y 2 ∂x ∂y If f ( z ) is analytic , then the Cauchy - Riemann relations are satisfied. ⇒ ∂f / ∂z * = 0 implies an analytic fonction of z contains the combination of x + iy , not x − iy
  • 6. If Cauchy - Riemann relations are satisfied ∂ ∂u ∂ ∂v ∂ ∂v ∂ ∂u ∂ 2u ∂ 2u (1) ( ) = ( )= ( ) = − ( )⇒ 2 + 2 2 = 0 ∂x ∂x ∂x ∂y ∂y ∂x ∂y ∂y ∂x ∂ y ∂ 2v ∂ 2v (2) the same result for function v ( x , y ) ⇒ 2 + 2 2 =0 ∂x ∂ y ⇒ u( x , y ) and v ( x , y ) are solutions of Laplace' s equation in two dimension. For two families of curves u( x , y ) = conctant and v ( x , y ) = constant, the normal vectors corresponding the two curves, respectively, are  ∂u ˆ ∂u ˆ  ∂v ˆ ∂v ˆ ∇u( x , y ) = i+ j and ∇v ( x , y ) = i+ j ∂x ∂y ∂x ∂y   ∂u ∂v ∂u ∂v ∂u ∂u ∂u ∂u ∇u ⋅ ∇v = + =− + = 0 orthogonal ∂x ∂x ∂y ∂y ∂x ∂y ∂y ∂x
  • 7. 20.3 Power series in a complex variable ∞ ∞ f (z) = ∑ an z n = ∑ an r n exp(inθ ) n= 0 n=0 ∞ if ∑ | an | r n is convergent ⇒ f ( z ) is absolutely convergent n=0 ∞ Is ∑ | a n | r n convergent or not, can be justisfied by" Cauchy root test". n= 0 1 The radius of convergence R ⇒ = lim | a n |1 / n ⇒ (1) | z |< R absolutely convergent R n→ ∞ (2) | z |> R divergent (3) | z |= R undetermined ∞ zn 1 (1) ∑ ⇒ lim ( )1 / n = 0 ⇒ R = ∞ converges for all z n= 0 n! n→∞ n! ∞ (2) ∑ n! z n ⇒ lim ( n! )1 / n = ∞ ⇒ R = 0 converges only at z = 0 n= 0 n →∞
  • 8. 20.4 Some elementary functions ∞ zn Define exp z = ∑ n = 0 n! Ex : Show that exp z1 exp z 2 = exp( z1 + z 2 ) ∞ ( z1 + z 2 ) n exp( z1 + z 2 ) = ∑ n=0 n! ∞ 1 n n = ∑ n!(C 0 z1 + C1n z1n−1 z2 + C 2 z1n−2 z2 + C rn z1n−r z2 + ... + C n z2 ) n 2 r n n n= 0 n s r Cr 1 n! 1 set n = r + s ⇒ the coeff. of z1 z 2 is = = n! n! ( n − r )! r ! s! r ! ∞ zs ∞ zr ∞ ∞ 1 s r exp z1 exp z 2 = ∑ ∑ = ∑∑ z1 z 2 s =0 s! r = 0 r ! s =0 r = 0 s! r ! s r There are the same coeff. of z1 z 2 for the above two terms.
  • 9. Define the complex comonent of a real number a > 0 ∞ z n (ln a ) n a = exp( z ln a ) = ∑ z n=0 n! (1) if a = e ⇒ e z = exp( z ln e ) = exp z the same as real number iy y 2 iy 3 (2) if a = e, z = iy ⇒ e = exp(iy ) = 1 − − + ... 2! 3! y2 y4 y3 = 1− + + ..... + i ( y − + ...) = cos y + i sin y 2! 4! 3! (3) if a = e , z = x + iy ⇒ e x + iy = e x e iy = exp( x )(cos y + i sin y )
  • 10. Set exp w = z Write z = r exp iθ for r is real and − π < θ ≤ π ⇒ z = r exp[i (θ + 2nπ )] ⇒ w = Lnz = ln r + i (θ + 2nπ ) Lnz is a multivalued function of z . Take its principal value by choosing n = 0 ⇒ ln z = ln r + iθ -π < θ ≤ π If t ≠ 0 and z are both complex numbers, we define t z = exp( zLnt ) Ex : Show that there are exactly n distinct nth roots of t . 1 1 tn = exp( Lnt ) and t = r exp[i (θ + 2kπ )] n 1 1 1 (θ + 2kπ ) (θ + 2kπ ) ⇒ tn = exp[ ln r + i ] = r n exp[i ] n n n
  • 11. 20.5 Multivalued functions and branch cuts A logarithmic function, a complex power and a complex root are all multivalued. Is the properties of analytic function still applied? Ex : f ( z ) = z 1 / 2 and z = r exp(iθ ) (A) C y (A) z traverse any closed contour C that dose not enclose the origin, θ return r to its original value after one complete θ x circuit. (B) y (B) θ → θ + 2π enclose the origin C' r 1/ 2 1/ 2 r exp(iθ / 2) → r exp[i (θ + 2π ) / 2] θ = − r 1 / 2 exp(iθ / 2) x ⇒ f (z) → − f (z) z = 0 is a branch point of the function f ( z ) = z 1 / 2
  • 12. Branch point: z remains unchanged while z traverse a closed contour C about some point. But a function f(z) changes after one complete circuit. Branch cut: It is a line (or curve) in the complex plane that we must cross , so the function remains single-valued. y Ex : f ( z ) = z 1 / 2 restrict θ ⇒ 0 ≤ θ < 2π ⇒ f ( z ) is single - valued 0 x
  • 13. Ex : Find the branch points of f ( z ) = z 2 + 1 , and hence sketch suitable arrangements of branch cuts. f ( z ) = z 2 + 1 = ( z + i )( z − i ) expected branch points : z = ± i set z − i = r1 exp(iθ1 ) and z + i = r2 exp(iθ 2 ) ⇒ f ( z ) = r1r2 exp(iθ1 / 2) exp(iθ 2 / 2) = r1r2 exp[i (θ1 + θ 2 )] If contour C encloses (1) neither branch point, then θ1 → θ1 , θ 2 → θ 2 ⇒ f ( z ) → f ( z ) (2) z = i but not z = − i , then θ1 → θ1 + 2π , θ 2 → θ 2 ⇒ f ( z ) → − f ( z ) (3) z = − i but not z = i , then θ1 → θ1 , θ 2 → θ 2 + 2π ⇒ f ( z ) → − f ( z ) (4) both branch points, then θ1 → θ1 + 2π ,θ 2 → θ 2 + 2π ⇒ f ( z ) → f ( z )
  • 14. f ( z ) changes value around loops containing either z = i or z = − i . We choose branch cut as follows : (A) y (B) y i i x x −i −i
  • 15. 20.6 Singularities and zeros of complex function g( z ) Isolated singularity (pole) : f ( z ) = ( z − z0 ) n n is a positive integer, g( z ) is analytic at all points in some neighborhood containing z = z0 and g( z0 ) ≠ 0, the f ( z ) has a pole of order n at z = z0 . * * An alternate definition for that f ( z ) has a pole of order n at z = z0 is lim [( z − z0 ) n f ( z )] = a z → z0 f ( z ) is analytic and a is a finite, non - zero complex number (1) if a = 0, then z = z0 is a pole of order less than n. (2) if a is infinite, then z = z0 is a pole of order greater than n. (3) if z = z0 is a pole of f ( z ) ⇒| f ( z ) |→ ∞ as z → z0 (4) from any direction, if no finite n satisfies the limit ⇒ essential singularity
  • 16. Ex : Find the singularities of the function 1 1 (1) f ( z ) = − 1− z 1+ z 2z ⇒ f (z) = poles of order 1 at z = 1 and z = −1 (1 − z )(1 + z ) (2) f ( z ) = tanh z sinh z exp z − exp(− z ) = = cosh z exp z + exp(− z ) f ( z ) has a singularity when exp z = − exp(− z ) ⇒ exp z = exp[i ( 2n + 1)π ] = exp(− z ) n is any integer 1 ⇒ 2 z = i ( 2n + 1)π ⇒ z = ( n + )πi 2 Using l' Hospital' s rule [ z − ( n + 1 / 2)πi ] sinh z [ z − ( n + 1 / 2)πi ] cosh z + sinh z lim { }= lim { }=1 z →( n+1 / 2 )πi cosh z z →( n+1 / 2 )πi sinh z each singularity is a simple pole (n = 1)
  • 17. Remove singularties : Singularity makes the value of f ( z ) undetermined, but lim f ( z ) z→ z0 exists and independent of the direction from which z0 is approached. Ex : Show that f ( z ) = sin z / z is a removable singularity at z = 0 Sol : lim f ( z ) = 0 / 0 undetermined z →0 1 z3 z5 z3 z5 f (z) = (z − + − ........) = 1 − + − .... z 3! 5! 3! 5! lim f ( z ) = 1 is independent of the way z → 0, so z →0 f ( z ) has a removable singularity at z = 0.
  • 18. The behavior of f ( z ) at infinity is given by that of f (1 / ξ ) at ξ = 0, where ξ = 1 / z Ex : Find the behavior at infinity of (i) f ( z ) = a + bz −2 (ii) f ( z ) = z (1 + z 2 ) and (iii) f ( z ) = exp z (i) f ( z ) = a + bz − 2 ⇒ set z = 1 / ξ ⇒ f (1 / ξ ) = a + bξ 2 is analytic at ξ = 0 ⇒ f ( z ) is analytic at z = ∞ (ii) f ( z ) = z (1 − z 2 ) ⇒ f (1 / ξ ) = 1 / ξ + 1 / ξ 3 has a pole of order 3 at z = ∞ ∞ (iii) f ( z ) = exp z ⇒ f (1 / ξ ) = ∑ (n! )−1ξ −n n= 0 f ( z ) has an essential singularity at z = ∞
  • 19. If f ( z0 ) = 0 and f ( z ) = ( z − z0 )n g ( z ), if n is a positive integer, and g( z0 ) ≠ 0 (i) z = z0 is called a zero of order n. (ii) if n = 1, z = z0 is called a simple zero. (iii) z = z0 is also a pole of order n of 1 / f ( z )
  • 20. 20.10 Complex integral y B A real continuous parameter t , for α ≤ t ≤ β C2 x = x ( t ), y = y( t ) and point A is t = α , C1 point B is t = β ⇒ ∫ f ( z )dz = ∫ ( u + iv )(dx + idy ) x C C A C3 = ∫ udx − ∫ vdy + i ∫ udy + i ∫ vdx C C C C β dx β dy β dy β dx =∫ u dt − ∫ v dt + i ∫ u dt + i ∫ v dt α dt α dt α dt α dt
  • 21. Ex : Evaluate the complex integral of f ( z ) = 1 / z , along the circle |z| = R, starting and finishing at z = R. y z ( t ) = R cos t + iR sin t , 0 ≤ t ≤ 2π C1 dx dy 1 x − iy R = − R sin t , = R cos t , f ( z ) = = 2 = u + iv , t dt dt x + iy x + y 2 x x cos t −y − sin t u= 2 = ,v = 2 = x + y2 R x + y2 R 1 2π cos t 2π − sin t ∫C1 z dz = ∫0 R (− R sin t )dt − ∫0 ( R ) R cos tdt 2π cos t 2π − sin t + i∫ R cos tdt + i ∫ ( )( − R sin t )dt 0 R 0 R = 0 + 0 + iπ + iπ = 2πi * * The integral is also calculated by dz 2π − R sin t + iR cos t 2π ∫C1 z 0 R cos t + iR sin t =∫ dt = ∫ idt = 2πi 0 The calculated result is independent of R.
  • 22. Ex : Evaluate the complex integral of f ( z ) = 1 / z along (i) the contour C 2 consisting of the semicircle | z |= R in the half - plane y ≥ 0 (ii) the contour C 3 made up of two straight lines C 3a and C 3b (i) This is just as in the previous example, but for y 0 ≤ t ≤ π ⇒ ∫ dz / z = πi iR C2 C 3b (ii) C 3a : z = (1 − t ) R + itR for 0 ≤ t ≤ 1 C 3a C 3b : − sR + i (1 − s ) R for 0 ≤ s ≤ 1 s=1 t=0 dz 1 − R + iR 1 − R − iR −R R x ∫C3 z 0 R + t (− R + iR) 0 iR + s(− R − iR) =∫ dt + ∫ dt 1 −1+ i 1 2t − 1 1 1 1st term ⇒ ∫ dt = ∫ dt + i ∫ dt 0 1 − t + it 0 1 − 2 t + 2t 2 0 1 − 2t + 2t 2 1 i t −1/ 2 1 = [ln(1 − 2t + 2t 2 )] |1 + [2 tan −1 ( 0 )] |0 2 2 1/ 2 i π π πi a x = 0 + [ − ( − )] = ∫ a2 + x2 dx = tan −1 ( ) + c 2 2 2 2 a
  • 23. 1 1+ i 1 (1 + i )[ s − i ( s − 1)] 2nd term ⇒ ∫ ds = ∫ ds 0 s + i ( s − 1) 0 2 s + ( s − 1) 2 1 2s − 1 1 1 =∫ ds + i ∫ ds 0 2s 2 − 2s + 1 0 2s 2 − 2s + 1 1 s −1/ 2 1 = [ln( 2 s 2 − 2 s + 1)] |1 + i tan −1 ( 0 ) |0 2 1/ 2 π π πi = 0 + i[ − ( − )] = 4 4 2 dz ⇒ ∫C3 z = πi The integral is independent of the different path.
  • 24. Ex : Evaluate the complex integral of f ( z ) = Re( z ) along the path C1 , C 2 and C 3 as shown in the previous examples. 2π (i) C1 : ∫ R cos t ( − R sin t + iR cos t )dt = iπR 2 0 π iπ 2 (ii) C 2 : ∫ R cos t ( − R sin t + iR cos t )dt = R 0 2 (iii) C 3 = C 3a + C 3b : 1 1 ∫0 (1 − t ) R( − R + iR )dt + ∫ ( − sR )( − R − iR )ds 0 1 1 = R 2 ∫ (1 − t )( −1 + i )dt + R 2 ∫ s(1 + i )ds 0 0 1 2 1 = R ( −1 + i ) + R 2 (1 + i ) = iR 2 2 2 The integral depends on the different path.
  • 25. 20.11 Cauchy theorem If f ( z ) is an analytic function, and f ' ( z ) is continuous at each point within and on a closed contour C ⇒ ∫ f ( z )dz = 0 C ∂p( x , y ) ∂q( x , y ) If and are continuous within and ∂x ∂y on a closed contour C, then by two - demensional ∂p ∂q divergence theorem ⇒ ∫∫ ( + )dxdy = ∫ ( pdy − qdx ) R ∂x ∂y C f ( z ) = u + iv and dz = dx + idy I = ∫ f ( z )dz = ∫ ( udx − vdy ) + i ∫ (vdx + udy ) C C C ∂ ( − u) ∂ ( − v ) ∂ ( − v ) ∂u = ∫∫ [ + ]dxdy + i ∫∫ [ + ]dxdy = 0 R ∂y ∂x R ∂y ∂x f ( z ) is analytic and the Cauchy - Riemann relations apply.
  • 26. Ex : Suppose twos points A and B in the complex plane are joined by two different paths C1 and C 2 . Show that if f ( z ) is an analytic function on each path and in the region enclosed by the two paths then the integral of f ( z ) is the same along C1 and C 2 . y B ∫C 1 f ( z )dz − ∫ C2 f ( z )dz = ∫ C1 − C 2 f ( z )dz = 0 C1 path C1 − C 2 forms a closed contour enclosing R R ⇒∫ f ( z )dz = ∫ f ( z )dz x C1 C2 A
  • 27. Ex : Consider two closed contour C and γ in the Argand diagram, γ being sufficiently small that it lies completely with C . Show that if the function f ( z ) is analytic in the region between the two contours then ∫ f ( z )dz = ∫ f ( z )dz C γ the area is bounded by Γ, and f ( z ) is analytic C y ∫Γ f ( z )dz = 0 γ C1 = ∫ f ( z )dz + ∫ f ( z )dz + ∫ f ( z )dz + ∫ f ( z )dz C γ C1 C2 If take the direction of contour γ as that of C2 contour C ⇒ ∫ f ( z )dz = ∫ f ( z )dz C γ x Morera' s theorem : if f ( z ) is a continuous function of z in a closed domain R bounded by a curve C , for ∫ f ( z )dz = 0 ⇒ f ( z ) is analytic. C
  • 28. 20.12 Cauchy’s integral formula If f ( z ) is analytic within and on a closed contour C 1 f (z) 2πi ∫C z − z0 and z0 is a point within C then f ( z0 ) = dz f (z) f (z) C I=∫ dz = ∫ dz C z − z0 γ z−z 0 for z = z0 + ρ exp(iθ ), dz = iρ exp(iθ )dθ γ 2π f ( z0 + ρe iθ ) z0 I=∫ iρe iθ dθ 0 ρe iθ 2π ρ →0 iθ = i∫ f ( z0 + ρe )dθ = 2πif ( z0 ) 0
  • 29. The integral form of the derivative of a complex function : 1 f (z) f ' ( z0 ) = 2πi ∫C ( z − z )2 dz 0 f ( z 0 + h) − f ( z 0 ) f ' ( z0 ) = lim h→ 0 h 1 f (z) 1 1 = lim [ h→0 2πi ∫C h z − z0 − h z − z0 )dz ] ( − 1 f (z) = lim [ h→ 0 2πi ∫C ( z − z0 − h)( z − z0 )dz ] 1 f (z) = 2πi ∫C ( z − z )2 dz 0 n! f (z) 2πi ∫C ( z − z0 )n+1 For nth derivative f ( n ) ( z0 ) = dz
  • 30. Ex : Suppose that f ( z ) is analytic inside and on a circleC of radius R centered on the point z = z0 . If | f ( z ) |≤ M on the circle, where Mn! M is some constant, show that | f ( n ) ( z0 ) |≤ n . R n! f ( z )dz n! M Mn! | f ( n ) ( z0 ) |= |∫ |≤ 2πR = n 2π C ( z − z0 )n+1 2π R n+1 R Liouville' s theorem : If f ( z ) is analytic and bounded for all z then f ( z ) is a constant. Mn! Using Cauchy' s inequality : | f ( n ) ( z0 ) |≤ Rn set n = 1 and let R → ∞ ⇒ | f ' ( z0 ) |= 0 ⇒ f ' ( z0 ) = 0 Since f ( z ) is analytic for all z , we may take z0 as any point in the z - plane. f ' ( z ) = 0 for all z ⇒ f ( z ) = constant
  • 31. 20.13 Taylor and Laurent series Taylor’s theorem: If f ( z ) is analytic inside and on a circle C of radius R centered on the point z = z0 , and z is a point inside C, then ∞ ∞ f ( n ) ( z0 ) f (z) = ∑ a n ( z − z0 ) n = ∑ n! ( z − z0 ) n n= 0 n= 0 1 f (ξ ) 2πi ∫C ξ − z f ( z ) is analytic inside and on C, so f ( z ) = dξ where ξ lies on C ∞ 1 z − z0 1 1 z − z0 n expand ξ −z as a geometric series in ξ − z0 ⇒ = ξ − z ξ − z0 ∑( ξ − z0 ) n=0 1 f (ξ ) ∞ z − z0 n 1 ∞ f (ξ ) ⇒ f (z) = ∑ ( ξ − z ) dξ = 2πi ∑ ( z − z0 )n ∫C (ξ − z )n+1 dξ 2πi ∫C ξ − z0 n= 0 0 n= 0 0 1 ∞ n 2πif ( n) ( z0 ) ∞ n f (n) ( z0 ) = ∑ 2πi n=0 ( z − z0 ) n! = ∑ ( z − z0 ) n! n=0
  • 32. If f ( z ) has a pole of order p at z = z0 but is analytic at every other point inside and on C . Then g ( z ) = ( z − z0 ) p f ( z ) is analytic at z = z0 and expanded as a Taylor ∞ series g( z ) = ∑ bn ( z − z0 )n . n=0 Thus , for all z inside C f ( z ) can be exp anded as a Laurent series a− p a − p +1 a f (z) = + + ....... + −1 + a0 + a1 ( z − z0 ) + a 2 ( z − z0 )2 ( z − z0 ) p ( z − z0 ) p−1 z − z0 g ( n ) ( z0 ) 1 g( z ) a n = bn+ p and bn = n! = 2πi ∫ ( z − z )n+1 dz C2 0 R 1 g( z ) 1 f (z) 2πi ∫ ( z − z0 ) n+1+ p 2πi ∫ ( z − z0 ) n+1 ⇒ an = dz = dz z 0 C1 ∞ f (z) = ∑ an ( z − z0 )n is analytic in a region R between n= −∞ two circles C1 and C 2 centered on z = z0
  • 33. f (z) = ∑ a n ( z − z0 ) n n = −∞ (1) If f ( z ) is analytic at z = z0 , then all an = 0 for n < 0. It may happen a n = 0 for n ≥ 0, the first non - vanishing term is a m ( z-z0 ) m with m > 0, f ( z ) is said to have a zero of order m at z = z0 . (2) If f ( z ) is not analytic at z = z0 (i) possible to find a − p ≠ 0 but a − p− k = 0 for all k > 0 f ( z ) has a pole of order p at z = z0 , a −1 is called the residue of f ( z ) (ii) impossible to find a lowest value of − p ⇒ essential singularity
  • 34. 1 Ex : Find the Laurent series of f ( z ) = 3 about the singularities z ( z − 2) z = 0 and z = 2. Hence verify that z = 0 is a pole of order 1 and z = 2 is a pole of order 3, and find the residue of f ( z ) at each pole. (1) point z = 0 −1 −1 − z ( −3)( −4) − z 2 ( −3)( −4)( −5) − z 3 f (z) = 3 = [1 + ( −3)( ) + ( ) + ( ) + ...] 8 z (1 − z / 2) 8z 2 2! 2 3! 2 1 3 3 5z 2 =− − − z− − ... z = 0 is a pole of order 1 8 z 16 16 32 (2) point z = 2 ⇒ set z − 2 = ξ ⇒ z( z − 2) 3 = ( 2 + ξ )ξ 3 = 2ξ 3 (1 + ξ / 2) 1 1 ξ ξ ξ ξ f (z) = 3 = 3 [1 − ( ) + ( )2 − ( )3 + ( )4 − ...] 2ξ (1 + ξ / 2) 2ξ 2 2 2 2 1 1 1 1 ξ 1 1 1 1 z−2 = − + − + − .. = − + − + − 2ξ 3 4ξ 2 8ξ 16 32 2( z − 2) 3 4( z − 2) 2 8( z − 2) 16 32 z = 2 is a pole of order 3, the residue of f ( z ) at z = 2 is 1 / 8.
  • 35. How to obtain the residue ? a− m a−1 f (z) = + ...... + + a0 + a1 ( z − z0 ) + a 2 ( z − z0 )2 + ... ( z − z0 ) m ( z − z0 ) ⇒ ( z − z0 )m f ( z ) = a − m + a − m +1 ( z − z0 ) + ....... + a −1 ( z − z0 )m −1 + ... d m −1 ∞ ⇒ m −1 [( z − z0 ) f ( z )] = ( m − 1)! a −1 + ∑ bn ( z − z0 )n m dz n =1 Take the limit z → z0 1 d m −1 R( z0 ) = a −1 = lim { [( z − z0 ) m f ( z )]} residue at z = z0 z → z0 ( m − 1)! dz m −1 (1) For a simple pole m = 1 ⇒ R( z0 ) = lim [( z − z0 ) f ( z )] z → z0 g( z ) (2) If f ( z ) has a simple at z = z0 and f ( z ) = , g ( z ) is analytic and h( z ) non - zero at z0 and h( z0 ) = 0 ( z − z0 ) g ( z ) ( z − z0 ) 1 g( z ) ⇒ R( z0 ) = lim = g ( z0 ) lim = g ( z0 ) lim ' = ' 0 z → z0 h( z ) z → z0 h( z ) z → z0 h ( z ) h ( z0 )
  • 36. Ex : Suppose that f ( z ) has a pole of order m at the point z = z0 . By considering the Laurent series of f ( z ) about z0 , deriving a general expression for the residue R( z0 ) of f ( z ) at z = z0 . Hence evaluate exp iz the residue of the function f ( z ) = 2 2 at the point z = i . ( z + 1) exp iz exp iz f (z) = 2 2 = 2 2 poles of order 2 at z = i and z = − i ( z + 1) (z + i) (z − i) for pole at z = i : d d exp iz i 2 [( z − i ) 2 f ( z )] = [ 2 ]= 2 exp iz − 3 exp iz dz dz ( z + i ) (z + i) (z + i) 1 i 2 −i R( i ) = [ e −1 − e −1 ] = 1! ( 2i ) 2 ( 2i ) 3 2e
  • 37. 20.14 Residue theorem f ( z ) has a pole of order m at z = z0 C ∞ γ f (z) = ∑ a n ( z − z0 ) n n= − m ρ • z0 I = ∫ f ( z )dz = ∫ f ( z )dz C γ set z = z0 + ρe iθ ⇒ dz = iρe iθ dθ ∞ ∞ 2π I= ∑ an ∫ ( z − z0 ) dz = C n ∑ an ∫ 0 iρ n+1e i ( n+1)θ dθ n= − m n= − m 2π n +1 i ( n+1)θ iρ n+1e i ( n+1)θ 2π for n ≠ −1 ⇒ ∫ iρ e dθ = |0 = 0 0 i ( n + 1) 2π for n = 1 ⇒ ∫ idθ = 2πi 0 I = ∫ f ( z )dz = 2πia −1 C
  • 38. Residue theorem: f ( z ) is continuous within and on a closed contour C and analytic, except for a finite number of poles within C ∫C f ( z )dz = 2πi ∑ R j j ∑ R j is the sum of the residues of f ( z ) at its poles within C j C C'
  • 39. The integral I of f ( z ) along an open contour C C if f ( z ) has a simple pole at z = z0 ρ ⇒ f ( z ) = φ ( z ) + a−1 ( z − z0 ) −1 z0 φ ( z ) is analytic within some neighbour surrounding z0 | z − z0 |= ρ and θ1 ≤ arg( z − z0 ) ≤ θ 2 ρ is chosen small enough that no singularity of f ( z ) except z = z0 I = ∫ f ( z )dz = ∫ φ ( z )dz + a −1 ∫ ( z − z0 ) −1 dz C C C lim ρ →0 C ∫ φ ( z )dz = 0 θ2 1 I = lim ∫ f ( z )dz = lim (a −1 ∫ iθ iρe iθ dθ ) = ia −1 (θ 2 − θ1 ) ρ →0 C ρ →0 ρe θ1 for a closed contour θ 2 = θ1 + 2π ⇒ I = 2πia −1
  • 40. 20.16 Integrals of sinusoidal functions 2π ∫0 F (cos θ , sin θ )dθ set z = exp iθ in unit circle 1 1 1 1 ⇒ cos θ = ( z + ), sin θ = ( z − ), dθ = − iz −1dz 2 z 2i z 2π cos 2θ Ex : Evaluate I = ∫ 2 2 dθ for b > a > 0 0 a + b − 2ab cos θ 1 n 1 cos nθ = ( z + z − n ) ⇒ cos 2θ = ( z 2 + z − 2 ) 2 2 1 2 1 cos 2θ ( z + z − 2 )( − iz −1 )dz − ( z 4 + 1)idz dθ = 2 = 2 2 a 2 + b 2 − 2ab cos θ 2 2 1 −1 z ( za 2 + zb 2 − abz 2 − ab ) a + b − 2ab ⋅ ( z + z ) 2 i ( z 4 + 1)dz i ( z 4 + 1) = = dz 2ab 2 2 a b 2ab 2 a b z ( z − z ( − + ) + 1) z ( z − )( z − ) b a b a
  • 41. i z4 + 1 2ab ∫C I= dz double poles at z = 0 and z = a / b within the unit circle a b z 2 ( z − )( z − ) b a 1 d m −1 Residue : R( z0 ) = lim { [( z − z0 ) m f ( z )] z → z0 ( m − 1)! dz m −1 (1) pole at z = 0, m = 2 1 d 2 z4 + 1 R(0) = lim { [z 2 ]} z →0 1! dz z ( z − a / b )( z − b / a ) 4z 3 ( z 4 + 1)( −1)[2 z − (a / b + b / a )] = lim { + }= a/b+b/a z →0 ( z − a / b )( z − b / a ) 2 ( z − a / b) ( z − b / a ) 2 (2) pole at z = a / b, m = 1 z4 + 1 (a / b)4 + 1 − (a 4 + b 4 ) R(a / b) = lim [( z − a / b ) 2 ]= 2 = z →a / b z ( z − a / b )( z − b / a ) (a / b) (a / b − b / a ) ab(b 2 − a 2 ) i a 2 + b2 a 4 + b4 2πa 2 I = 2πi × [ − ]= 2 2 2ab ab ab(b − a ) b (b − a 2 ) 2 2
  • 42. 20.17 Some infinite integrals ∞ ∫−∞ f ( x )dx f ( z ) has the following properties : (1) f ( z ) is analytic in the upper half - plane, Im z ≥ 0, except for a finite number of poles, none of which is on the real axis. (2) on a semicircle Γ of radius R, R times the maximum of y | f | on Γ tends to zero as R → ∞ (a sufficient condition is that zf ( z ) → 0 as | z |→ ∞ ). Γ 0 ∞ (3) ∫ f ( x )dx and ∫0 f ( x )dx both exist −∞ x ∞ ⇒ ∫− ∞ f ( x )dx = 2π i ∑ R j −R 0 R j for | ∫ f ( z )dz |≤ 2π R × (maximum of | f | on Γ ), the integral along Γ Γ tends to zero as R → ∞ .
  • 43. dx Ex : Evaluate I = ∫0 2 (x + a ) 2 4 a is real dz R dx dz ∫C ( z 2 + a 2 )4 = ∫− R ( x 2 + a 2 )4 ∫Γ ( z 2 + a 2 )4 + as R → ∞ Γ dz dz ∞ dx ai ⇒∫ →0⇒ ∫C ( z 2 + a 2 )4 = ∫− ∞ ( x 2 + a 2 )4 Γ ( z 2 + a 2 )4 ( z 2 + a 2 )4 = 0 ⇒ poles of order 4 at z = ± ai , −R 0 R only z = ai at the upper half - plane 1 1 1 iξ − 4 set z = ai + ξ , ξ → 0 ⇒ 2 2 4 = 2 4 = 4 (1 − ) (z + a ) ( 2aiξ + ξ ) ( 2aiξ ) 2a 1 ( −4)( −5)( −6) − i 3 − 5i the coefficient of ξ −1 is ( ) = ( 2a ) 4 3! 2a 32a 7 ∞ dx − 5i 10π 1 10π 5π ∫0 ( x 2 + a 2 )4 = 2πi ( 32a 7 )= 32a 7 ⇒I = × 2 32a 7 = 32a 7
  • 44. For poles on the real axis: y Γ Principal value of the integral, defined as ρ → 0 R z0 − ρ R P∫ f ( x )dx = ∫− R f ( x )dx + ∫z + ρ f ( x )dx γ ρ −R 0 -R 0 z0 R x for a closed contour C z0 − ρ R ∫C f ( z )dz = ∫− R f ( x )dx + ∫γ f ( z )dz + ∫z + ρ 0 f ( x )dx + ∫Γ f ( z )dz R = P∫ f ( x )dx + ∫γ f ( z )dz + ∫Γ f ( z )dz −R (1) for ∫ f ( z )dz has a pole at z = z0 ⇒ ∫ f ( z )dz = −πia1 γ γ iθ (2) for ∫ f ( z )dz set z = Re dz = i Re iθ dθ Γ ⇒ ∫Γ f ( z )dz = ∫Γ f (Re iθ )i Re iθ dθ If f ( z ) vanishes faster than 1 / R 2 as R → ∞, the integral is zero
  • 45. Jordan’s lemma (1) f ( z ) is analytic in the upper half - plane except for a finite number of poles in Im z > 0 ( 2) the maximum of | f ( z ) |→ 0 as | z |→ ∞ in the upper half - plane (3) m > 0, then ∫Γ e imz IΓ = f ( z )dz → 0 as R → ∞, Γ is the semicircular contour for 0 ≤ θ ≤ π / 2, 1 ≥ sin θ / θ ≥ π / 2 f (θ ) f = 2θ / π | exp(imz )| = | exp(− mR sin θ )| π IΓ ≤ ∫Γ |e imz f ( z )||dz| ≤ MR ∫ e − mR sin θ dθ 0 f = sin θ π/ 2 − mR sin θ = 2 MR ∫ e dθ 0 M is the maximum of |f ( z )| on |z| = R, R → ∞ M → 0 π /2 θ πM π / 2 − mR ( 2θ / π ) πM I Γ < 2 MR ∫ e (1 − e − mR ) < dθ = 0 m m as R → ∞ ⇒ M → 0 ⇒ I Γ → 0
  • 46. cos mx ∞ Ex : Find the principal value of ∫− ∞ x − a dx a real, m > 0 Γ e imz Consider the integral I = ∫C z−a dz = 0 no pole in the γ ρ −1 upper half - plane, and |( z − a ) | → 0 as |z| → ∞ -R 0 a R e imz I = ∫C z−a dz a−ρ e imx e imz R e imx e imz = ∫− R x−a dx + ∫γ z−a dz + ∫a + ρ x−a dx + ∫Γ z−a dz = 0 e imz As R → ∞ and ρ → 0 ⇒ ∫ dz → 0 Γ z−a ∞e imx ⇒ P∫ dx − iπa−1 = 0 and a−1 = e ima −∞ x − a ∞ cos mx ∞ sin mx ⇒ P∫ dx = −π sin ma and P ∫ dx = π cos ma −∞ x − a −∞ x − a
  • 47. 20.18 Integral of multivalued functions 1/ 2 y Multivalued functions such as z , Lnz Single branch point is at the otigin. We let R → ∞ R Γ and ρ → 0. The integrand is multivalued, its values γ along two lines AB and CD joining z = ρ to z = R A B are not equal and opposite. ρ x C D ∞ dx Ex : I = ∫0 ( x + a )3 x1 / 2 for a > 0 (1) the integrand f ( z ) = ( z + a ) −3 z −1 / 2 , |zf ( z )| → 0 as ρ → 0 and R → ∞ the two circles make no contribution to the contour integral (2) pole at z = − a , and ( − a )1 / 2 = a 1 / 2 e iπ / 2 = ia 1 / 2 1 d 3 −1 1 R( − a ) = lim [( z + a )3 ] z → − a ( 3 − 1)! dz 3 − 1 3 1/ 2 (z + a) z 1 d 2 −1 / 2 − 3i = lim z = z → − a 2! dz 2 8a 5 / 2
  • 48. − 3i ∫AB dz + ∫Γ dz + ∫DC dz + ∫γ dz = 2πi ( 8a 5/2 ) and ∫ dz = 0 and ∫γ dz = 0 Γ along line AB ⇒ z = xe i 0 , along line CD ⇒ z = xe i 2π ∞ dx 0 dx 3π ∫0, A→ B ( x + a )3 x1 / 2 ∞,C → D ( xe i 2π + a )3 x1 / 2e (1 / 2×2πi ) 4a 5 / 2 +∫ = 1 ∞ dx 3π ⇒ (1 − iπ )∫ = e 0 ( x + a )3 x1 / 2 4a 5 / 2 ∞ dx 3π ⇒∫ = 0 ( x + a )3 x1 / 2 8a 5 / 2
  • 49. x sin x Ex : Evaluate I (σ ) = ∫− ∞ x 2 − σ 2 dx z sin z 1 ze iz 1 ze − iz ∫C z2 − σ 2 dz = 2i ∫C 1 z2 − σ 2 dz − 2i ∫C 2 2 z −σ 2 dz = I1 + I 2 (1) for I1 , the contour is choosed on the upper half - plane C1 due to the term e iz , and only one pole at z = σ . Γ 1 ze iz 1 −σ − ρ xe ix γ1 I1 = 2i ∫C 1 z2 − σ 2 dz = 2i ∫− R x2 − σ 2 dx ρ 1 σ −ρ xe ix 1 ∞ xe ix -R -σ σ R + 2i ∫−σ + ρ x2 − σ 2 dx + 2i ∫σ + ρ x 2 − σ 2 dx γ2 1 ze iz 1 ze iz 1 ze iz + 2i ∫γ 1 z2 − σ 2 dz + ∫ 2 2i γ 2 z − σ 2 dz + 2i ∫Γ z 2 − σ 2 dz 1 σe iσ π = 2πi × Res( z = σ ) = π = e iσ 2i 2σ 2
  • 50. As ρ → 0 and R → ∞ ⇒ ∫ dz → 0 Γ 1 ze iz 1 − π − iσ 2i ∫γ 1 ( z + σ )( z − σ ) dz = 2i × ( −πi )Res( z = −σ ) = 4 e 1 ze iz 1 π ∫γ dz = × πiRes( z = σ ) = e iσ 2i 2 ( z + σ )( z − σ ) 2i 4 1 ∞ xe ix π π γ1 dx + (e iσ − e − iσ ) = e iσ 2i ∫− ∞ x 2 − σ 2 I1 = σ 4 2 −σ (2) for I 2 , we choose the lower half - plane by the − iz Γ γ2 term e , only one pole at z = −σ −1 ze − iz − 1 −σ − ρ xe − ix I2 = 2i ∫C2 z 2 − σ 2 dz = 2i ∫− R x 2 − σ 2 dx C2 1 σ −ρ xe − ix 1 ∞ xe − ix 1 ze − iz − 2i ∫−σ + ρ x2 − σ 2 dx − 2i ∫σ + ρ x2 − σ 2 dx − 2i ∫γ 1 2 z −σ 2 dz 1 ze − iz 1 ze − iz −1 ( −σ )e iσ π − ∫γ dz − ∫Γ dz = ( ) × ( −2πi ) = e iσ 2i 2 z2 − σ 2 2i z2 − σ 2 2i − 2σ 2
  • 51. As ρ → 0, R → ∞ ⇒ ∫ dz → 0 Γ −1 ze − iz −1 ( −σ )e iσ π = e iσ 2i ∫γ 1 ( z + σ )( z − σ ) dz = ( )( −πi ) 2i − 2σ 4 −1 ze − iz −1 σe − iσ − π − iσ 2i ∫γ 2 ( z + σ )( z − σ )dz = ( 2i )(πi ) 2σ = 4 e − 1 ∞ xe − ix π π dx + (e iσ − e − iσ ) = e iσ 2i ∫− ∞ x 2 − σ 2 I2 = 4 2 − 1 ∞ xe − ix π 1 dx = e iσ − (e iσ − e − iσ ) 2i ∫− ∞ x 2 − σ 2 ⇒ 2 4 ∞ x sin x 1 ∞ xe ix 1 ∞ xe − ix I (σ ) = ∫ 2i ∫− ∞ x 2 − σ 2 2i ∫− ∞ x 2 − σ 2 dx = dx − dx −∞ x 2 − σ 2 π π π π = e iσ − ( e iσ − e − iσ ) + e iσ − ( e iσ − e − iσ ) 2 4 2 4 π π π = πe iσ − e iσ + e − iσ = (e iσ + e − iσ ) = π cos σ 2 2 2